30
MINISTRY OF PUBLIC HEALTH OF UKRAINE Department of human resources policy, education and science Testing Board Test items for licensing examination Krok 2 MEDICINE Student ID Last name Variant ___________________

Department of human resources policy, education …D. Pelvioperitonitis E. Incipient abortion 5. 5 weeks after hypothermia a 22-year-old patient developed fever, weakness, muscle pain,

  • Upload
    others

  • View
    5

  • Download
    0

Embed Size (px)

Citation preview

Page 1: Department of human resources policy, education …D. Pelvioperitonitis E. Incipient abortion 5. 5 weeks after hypothermia a 22-year-old patient developed fever, weakness, muscle pain,

MINISTRY OF PUBLIC HEALTH OF UKRAINE

Department of human resources policy, education and science

Testing Board

Test items for licensing examination

Krok 2MEDICINE

Student ID Last name

Variant ___________________

Page 2: Department of human resources policy, education …D. Pelvioperitonitis E. Incipient abortion 5. 5 weeks after hypothermia a 22-year-old patient developed fever, weakness, muscle pain,

Authors of items: Ahafonova O.O., Ambrozevych Z.M., Barannyk S.I., Berezniuk V.V., Berezov V.M.,Bilenko O.A., Bilyk O.V., Bilyk V.D., Blikhar V.Ye., Bolbot Yu.K., Borzova O.Yu., Bukhtieieva E.R.,Bukhtiyarova O.H., Buriak V.M., Butina L.I., Butvyn I.M., Chaika O.O., Chekanov S.L., Chelpan L.L.,Chonka I.I., Chuiko Yu.M., Dankyna I.A., Dashchuk A.M., Demchenko T.V., Desiatska Yu.V., Drin T.M.,Duplenko P.Yu., Dzis N.P., Filippova O.Yu., Franchuk O.A., Hahara V.F., Havrylova L.O., Henyk N.I.,Herasymenko O.I., Hovalenkova O.L., Hrydasova V.D., Hubka O.V., Hyrla Ya.V., Kalinovska I.V.,Kaliy V.V., Kalyberdenko V.B., Kandyba V.P., Karliychuk O.O., Kirieieva T.V., Klymenko A.V.,Koliush O.I., Kompaniyets K.M., Kondratenko P.H., Kondratiev V.O., Konopkina L.I., Konovalova N.V.,Kopchak O.V., Korovka S.Ya., Kovalchuk P.Ye., Kovalenko S.V., Kovtunenko R.V., Krut Yu.Ya.,Kryvenko Z.F., Kryvonosov M.V., Kryzhanovsky Yu.M., Kudria V.I., Lavrinchuk I.O., Lakusta N.M.,Lebediuk M.M., Leshchenko K.A., Liulka O.M., Makieieva N.I., Malovany V.V., Marushko Yu.V.,Martsynik Ye.M., Melnychuk L.V., Mierkulova N.F., Mitiunina N.I., Miziuk V.V., Moroz I.M.,Muravska O.M., Murtazin L.M., Mysak A.I., Nadraha O.B., Nechytailo Yu.M., Nerianov K.Yu.,Nikolaichuk O.M., Nishkumai O.I., Nykoniuk T.R., Olifierovska R.P., Parashchuk Yu.S., Pashchenko I.V.,Pertseva N.O., Petrynych V.V., Pisotska L.A., Pohorelov O.V., Proniv L.M., Pryshliak O.Ya.,Radchenko O.M., Reitmaier M.Y., Riapolova T.L., Romanenko V.N., Rudai V.V., Rudenko M.M.,Rudenko S.M., Ruzhytska O.O., Samardakova H.O., Semeniak A.V., Semenukha K.V., Sennikov I.A.,Serheta I.V., Shapkin V.Ye., Shkrobanets I.D., Shorikov Ye.I., Shusterman T.Y., Shvyhar L.V.,Sidykh N.M., Sierkova V.K., Sikorska M.V., Smachylo I.V., Smoliak L.L., Snizhko S.S., Soboleva N.P.,Sokolov O.B., Soldak I.I., Sonnyk H.T., Sorochan V.V., Sotnik Yu.P., Suk V.H., Svyrydova V.V.,Svystunov I.V., Sylenko H.Ya., Sychova V.V., Synoverska O.B., Tarasova V.I., Teliushchenko O.D.,Tiuieva N.V., Todoriko L.D., Tokariev A.V., Tonkohlas O.A., Tretiakevych Z.M., Troian V.I.,Tykhonova S.A., Ukhal M.I., Vankhanen N.V., Vatanska I.Yu., Volianska A.H., Voloshyna L.O.,Vorokhta Yu.M., Vykhovanets T.A., Yakovenko I.K., Yermachenko T.P., Yevtushenko V.V.,Yurchenko I.V., Yutanova A.V., Zakharov V.K., Zheliba M.D., Zoria A.V. and Committees of professionalexpertise

Item reviewers. Agafonova O.O., Anisimov Ye.M., Bab’yak T.Ye., Chursina T.Ya., Dyndar O.A.,Grygorov Yu.B., Grynzovs’ky A.M., Gubka V.O., Gutsalenko O.O., Kalinina S.Yu., Karapetyan K.G.,Khrapach V.V., Kolesnyk O.M., Kolosovych I.V., Kondratyuk V.Ye., Kopcha V.S., Kravchenko O.V.,Kutovy O.B., Kuz’mina I.Yu., Malanchuk L.M., Martynyuk L.P., Mishchenko V.P., Muravs’ka O.M.,Petrushenko V.V., Prokhorova M.P., Pryshlyak O.Ya., Puzanova O.G., Pyptyuk O.V., Shestakova I.V.,Shevtsova T.I., Stovban I.V., Tsvirenko S.M., Usenko S.G., Vakal’uk I.P., Volyans’ka A.G.

The book includes test items for use at licensing integrated examination “Krok 2. Medicine” andfurther use in teaching.

The book has been developed for students of medical, pediatric and medical-and-prophylacticfaculties and academic staff of higher medical educational establishments.

Approved by Ministry of Public Health of Ukraine as examination and teachingpublication based on expert conclusions (Orders of MPH of Ukraine of14.08.1998 №251, of 27.12.1999 №303, of 16.10.2002 №374, of 29.05.2003 №233).

© Copyright Testing Board.

General InstructionEvery one of these numbered questions or unfinished statements in

this chapter corresponds to answers or statements endings. Choose theanswer (finished statements) that fits best and fill in the circle with thecorresponding Latin letter on the answer sheet.

Page 3: Department of human resources policy, education …D. Pelvioperitonitis E. Incipient abortion 5. 5 weeks after hypothermia a 22-year-old patient developed fever, weakness, muscle pain,

Крок 2 Medicine (англомовний варiант, iноземнi студенти) 2017 рiк 1

1. A 32-year-old welder complains ofweakness and fever. His illness initiallypresented as tonsillitis one month earlier. Onexamination: body temperature - 38,9oC , RR-24/min., HR- 100/min., BP- 100/70 mm Hg,hemorrhages on the legs, enlargement of thelymph nodes. Complete blood count: Hb- 70g/l, RBC- 2, 2 · 1012/l, WBC- 3, 0 · 109/l wi-th 32% of blasts, 1% of eosinophils, 3% ofband neutrophils, 36% of segments, 20% oflymphocytes, and 8% of monocytes, ESR- 47mm/hour. What is the cause of anemia?

A. Acute leukemiaB. Chronic lympholeukemiaC. Aplastic anemaD. B12-deficient anemiaE. Chronic hemolytic anemia

2. After a 5-day-long celebration of hisdaughter’s wedding a 65-year-old patient”saw” in his yard many cats, chickens, andrats. He tried to chase them away, but wasscared off when the animals started to scoldhim and tried to harm him. Make the di-agnosis:

A. Delirium tremensB. Senile psychosisC. SchizophreniaD. Organic brain syndromeE. Reactive hallucinosis

3. An 8-year-old boy developed a temperatureof 37, 5oC two days after his recovery fromthe case of URTI. He complains of suffocati-on, heart pain. Objectively: the skin is pale,tachycardia, the I heart sound is weakened,short systolyc murmur in the 4th intercostalarea near the left edge of the breastbone.What heart disorder such clincal presentati-on is characteristic of?

A. Nonrheumatic myocarditisB. Primary rheumatic carditisC. MyocardiodystrophyD. Fallot’s tetradE. Cardiomyopathy

4. A 28-year-old woman has been deliveredto a hospital with acute pain in the lowerabdomen. There was a brief syncope. Thedelay of menstruation is 2 months. Objecti-vely: the patient has pale skin, BP- 90/50 mmHg, Ps- 110/min. Lower abdomen is extremelypainful. Vaginal examination reveals uterusenlargement. Promtov’s sign (pain during bi-manual gynecological examination) is posi-tive. Right uterine appendages are enlargedand very painful. Posterior vault hangs over.What is the most likely diagnosis?

A. Right-sided tubal pregnancyB. Right ovary apoplexyC. Acute right-sided salpingoophoritisD. PelvioperitonitisE. Incipient abortion

5. 5 weeks after hypothermia a 22-year-old patient developed fever, weakness,muscle pain, inability to move independently.Objectively: tenderness, induration ofshoulder and shin muscles, restricted activemovements, erythema on the anterior surfaceof the chest. There is a periorbital edemawith heliotropic erythema. Gottron’s sign ispresent. What investigation is required for thediagnosis verification?

A. Muscle biopsyB. Aminotransferase activityC. PneumoarthrographyD. ASO titerE. Rheumatoid factor

6. A 37-year-old woman complains ofheadaches, nausea, vomiting, spasms. Theonset of the disease occurred the day beforedue to her overexposure to cold. Objecti-vely: fever up to 40oC ; somnolence; rigidneck; Kernig’s symptom is positive on theboth sides; general hyperesthesia. Blood test:leucocytosis, increased ESR. Cerebrospinalfluid is turbid, yellow-tinted. What changes ofthe cerebrospinal fluid are most likely?

A. Neutrophilic pleocytosisB. Lymphocytic pleocytosisC. Blood in the cerebrospinal fluidD. Xanthochromia in the cerebrospinal fluidE. Albuminocytological dissociation

7. A 44-year-old patient complains of diffi-cult urination, sensation of incomplete uri-nary bladder emptying. Sonographic exami-nation of the urinary bladder near theurethra entrance revealed an oval well-defined hyperechogenic formation 2x3 cmlarge that was changing its position during theexamination. What conclusion can be made?

A. StoneB. Malignant tumour of the urinary bladderC. Urinary bladder polypD. Prostate adenomaE. Primary ureter tumour

8. 4 days after a patient received a gunshotwound to the soft tissues of middle thirdof the thigh, his condition suddenly begandeteriorating. There are complaints of bursti-ng pain in the wound; pain has been increasi-ng during the last 12 hours. Edema of skinand hypodermic tissue quickly grows. Bodytemperature is 38,2oC , heart rate is 102/min.The wound edges gape, are dull in color;the muscles, viable as of day before, nowprotrude into the wound, look boiled, are dullin color, have dirty-gray coating, and fall apartwhen held with forceps. What infection hasdeveloped in the wound?

Page 4: Department of human resources policy, education …D. Pelvioperitonitis E. Incipient abortion 5. 5 weeks after hypothermia a 22-year-old patient developed fever, weakness, muscle pain,

Крок 2 Medicine (англомовний варiант, iноземнi студенти) 2017 рiк 2

A. AnaerobicB. Aerobic gram-negativeC. PutridD. Aerobic gram-positiveE. Diphtheria of the wound

9. A 35-year-old patient has been sufferi-ng from an illness for 3 days. 5 days ago hereturned from a trip to Africa. The onset ofdisease was accompanied by fever up to 40oC ,chills, acute headache, myalgia. In the axi-llary region the lymph node enlarged up to3x6 cm can be palpated. The lymph node isdense, intensely painful, slightly mobile, wi-thout clear margins; the skin over the nodeis hyperenic and tight. Tachycardia is present.Make the preliminary diagnosis:

A. PlagueB. SepsisC. TularemiaD. LymphadenitisE. Anthrax

10. A parturient woman complains of pain inher mammary gland. In the painful area thereis an infiltration 3x4 cm in size with softenedcenter. Body temperature is 38,5oC . What isthe most likely diagnosis?

A. Acute suppurative mastitisB. PneumoniaC. PleurisyD. Milk retentionE. Birth trauma

11. A 52-year-old patient, who has beensuffering from angina pectoris, for 2 weekshas increasingly frequent pain attacks in thearea behind his sternum and his need for ni-troglycerine has increased. Objectively: thecondition is of moderate severity. The skin ispale. Heart sounds are weakened, rhythmic.Heart rate is 84/min. ECG shows no signs offocal myocardial injury. What is the most li-kely diagnosis?

A. Progressive angina pectorisB. First-time angina pectorisC. Stable NYHA functional class II anginapectorisD. Variant angina pectorisE. Acute cardiac infarction

12. A 9-year-old boy has been sufferingfrom multiple bronchiectasis since he was 3years old. Exacerbations occur frequently (3-4 times a year), after conservative therapythere are short remission periods. The di-sease progresses, the child is physicallyunderdeveloped, presents with pale skin,acrocyanosis, deformed nail plates in theshape of ”clock-face”. Bronchography revealssaccular bronchiectases in the lower lobe ofthe right lung. What further treatment tacticsshould be chosen?

A. Surgical interventionB. Continuation of conservative therapyC. PhysiotherapyD. Sanatorium-and-spa treatmentE. Physical training

13. Caries morbidity rate is 89% among resi-dents of a community. It is determined thatfluorine content in water is 0,1 mg/l. Whatpreventive measures should be taken?

A. Water fluorinationB. Tooth brushingC. Fluorine inhalationsD. Sealant applicationE. Introduction of more vegetables to the diet

14. A patient received flame burns of bothhands. On the dorsal and palmar surface ofthe hands there are blisters filled with serousfluid. The wrist joint region is hyperemic. Theforearms were not injured. What is the provi-sional diagnosis?

A. II degree flame burn of the hands, 4% ofbody surface areaB. II degree flame burn of the hands, 2% ofbody surface areaC. IIIa degree flame burn of the hands, 4% ofbody surface areaD. III degree flame burn of the hands, 4% ofbody surface areaE. IIb degree flame burn of the hands, 2% ofbody surface area

15. During the periodic medical examinationan assembly fitter (works on soldering detai-ls) didn’t report any health problems. Closerexamination revealed signs of asthenic-vegetative syndrome. Blood included redblood cells with basophilic aggregationsand a somewhat higher number of reti-culocytes, urine had a high concentration ofdelta-aminolevulinic acid. The complex ofsymptoms indicates the initial stage of chronicintoxication with:

A. LeadB. ManganeseC. MercuryD. TinE. Ethanol

16. A 29-year-old woman came to agynecologist with complaints of irritabili-ty, tearfulness, headache, nausea, occasionalvomiting, pain in the heart area, tachycardiaattacks, memory impairment, meteorism.These signs appear 6 days before menstruati-on and disappear the day before menstruationor during its first 2 days. On vaginal exami-nation: the uterus and uterine appendagesare without alterations. What diagnosis is themost likely?

Page 5: Department of human resources policy, education …D. Pelvioperitonitis E. Incipient abortion 5. 5 weeks after hypothermia a 22-year-old patient developed fever, weakness, muscle pain,

Крок 2 Medicine (англомовний варiант, iноземнi студенти) 2017 рiк 3

A. Premenstrual syndromeB. AlgodismenorrheaC. Ovarian apoplexyD. Genital endometriosisE. Neurosis

17. Examination of a group of persons livingon the same territory revealed the followingcommon symptoms: dark-yellow pigmentati-on of the tooth enamel, diffuse osteoporosisof bone apparatus, ossification of ligamentsand joints, functional disorders of the centralnervous system. This condition may be causedby the excessive concentration of the followi-ng microelement in food or drinking water:

A. FluorineB. CopperC. NickelD. IodineE. Cesium

18. An infant has been born at the 41st weekof gestation. The pregnancy was complicatedwith severe gestosis of the second semester.The weight of the baby is 2400 g, the height is50 cm. Objectively: the skin is flabby, the layerof subcutaneous fat is thin, hypomyotoniais observed, neonatal reflexes are weak.The internal organs are without pathologicalterations. This newborn can be assessed asa:

A. Full-term infant with prenatal growthretardationB. Premature infantC. Immature infantD. Postmature infantE. Full-term infant with normal body weight

19. A patient suffering from infiltrati-ve pulmonary tuberculosis was prescribedstreptomycin, rifampicin, isoniazid, pyrazi-namide, vitamin C . One month after the begi-nning of the treatment the patient startedcomplaining of reduced hearing and tinnitus.What drug has such a side effect?

A. StreptomycinB. IsoniazidC. RifampicinD. PyrazinamideE. Vitamin C

20. A 39-year-old man complains of morni-ng headaches, appetite loss, nausea, morni-ng vomiting, periodic nasal hemorrhages. Thepatient had a case of acute glomerulonephritisat the age of 15. Examination revealed rise ofarterial pressure up to 220/130 mm Hg, skinhemorrhages on his arms and legs, pallor ofskin and mucous membranes. What biochemi-cal parameter is the most important for maki-ng diagnosis in this case?

A. Blood creatinineB. Blood bilirubinC. Blood sodiumD. Uric acidE. Fibrinogen

21. A worker of a glass-blowing workshopcomplains of headache, irritability, visualimpairment - he sees everything as if througha ”net”. Objectively: hyperemic sclera, thi-ckened cornea, decreased opacity of pupi-ls, visual acuity is 0,8 in the left eye, 0,7 inthe right eye. The worker uses no means ofpersonal protection. What is the most likelydiagnosis?

A. CataractB. ConjunctivitisC. KeratitisD. BlepharospasmE. Progressive myopia

22. For a week a 42-year-old patient has beensuffering from fever attacks followed by hightemperature, which occur each 48 hours. Bodytemperature raises up to 40oC and decreasesin 3-4 hours with excessive sweating. The pati-ent presents with loss of appetite and generalfatigue. The skin is pale and sallow. The li-ver and spleen are enlarged and dense onpalpation. What method of diagnosis verifi-cation would be most efficient?

A. Microscopy of blood smear and thick bloodfilmB. Complete blood countC. Bacteriological analysisD. Immune-enzyme assayE. Microscopy of hanging blood drop

23. A 28-year-old patient is a drug addict. Hehas been sick for a year, when noticed generalweakness, increased sweating, and weightloss. He often had cases of respiratory di-seases. Within the last 2 days he demonstratesintermittent fever with profuse night sweati-ng, increased general weakness, developed di-arrhea with mucus and blood admixtures. Onexamination: polylymphadenopathy, herpeticrashes in the oral cavity; on abdominalpalpation: the liver and spleen are enlarged.What is the most likely diagnosis?

A. HIV-infectionB. Herpetic stomatitisC. Chronic lymphatic leukemiaD. Colon cancerE. Chronic sepsis

24. A 40-year-old patient has acute onsetof disease caused by overexposure to cold.Temperature has increased up to 39oC .Foul-smelling sputum is expectorated duri-ng coughig. Various moist crackles can beauscultated above the 3rd segment on the ri-ght. Blood test: leukocytes - 15, 0 · 109/l, stabneutrophils - 12%, ESR- 52 mm/hour. On X-ray: in the 3rd segment on the right there is a

Page 6: Department of human resources policy, education …D. Pelvioperitonitis E. Incipient abortion 5. 5 weeks after hypothermia a 22-year-old patient developed fever, weakness, muscle pain,

Крок 2 Medicine (англомовний варiант, iноземнi студенти) 2017 рiк 4

focus of shadow 3 cm in diameter, low density,with fuzzy smooth margins and a clearing inits center. What disease is most likely in thegiven case?

A. Pneumonia complicated by an abscessB. Infiltrative tuberculosisC. Peripheral pulmonary cancerD. Cystic echinococcosisE. Pulmonary cyst

25. A 48-year-old patient was found to havediffuse enlargement of the thyroid gland,exophthalmia, weight loss of 4 kg in 2 months,sweating. Objectively: HR- 105/min, BP-140/70 mm Hg. Defecation act is normal.What kind of therapy is recommended in thiscase?

A. MercazolilB. Radioactive iodineC. PropranololD. Lugol’s solutionE. Thyroxine

26. A 26-year-old woman is suspected tosuffer from systemic lupus erythematosus dueto systemic lesions of skin, vessels, joints,serous tunics, and heart that developed afterphotosensitization. The following is detected:LE cells, antibodies to native DNA, isolatedanti-centromere antibodies, rheumatoidfactor is 1:100, Wassermann reaction is posi-tive, circulating immune complex is 120 uni-ts. What immunological indicators are consi-dered to be specific to this disease?

A. DNA antibodiesB. Rheumatoid factorC. Anti-centromere antibodiesD. Immunoglobulin AE. Increased circulating immune complex

27. A woman came to a doctor with complai-nts of increased body temperature up to37,8oC and moderately sore throat for the last3 days. Objectively: mandibular lymph nodesare enlarged up to 3 cm. Palatine tonsils arehypertrophied, with gray coating that spreadsto the uvula and anterior pillars of the fauces.What is the most likely diagnosis?

A. Oropharyngeal diphtheriaB. Infectious mononucleosisC. Pseudomembranous (Vincent’s) tonsillitisD. AgranulocytosisE. Oropharyngeal candidiasis

28. A primigravida at the term of 20 weekscomplains of pain in her lower abdomen,smearing blood-streaked discharge from thegenital tracts. Uterine tone is increased, fetusis mobile. On vaginal examination: the uterusis enlarged according to the term, uterinecervix is shortened to 0,5 cm, external cervicalorifice is open by 2 cm. What is the most likelydiagnosis?

A. Risk of late abortion with hemorrhageB. Risk of late abortion without hemorrhageC. The process of late abortionD. Incomplete late abortionE. Attempted late abortion

29. A 65-year-old man was diagnosed withB12-deficient anemia and the treatment wasprescribed. A week later control blood testwas performed. What would be the early indi-cator of the therapy effectiveness?

A. Increased number of reticulocytesB. Increased hemoglobin levelC. Megaloblastic hematopoiesisD. Normoblastic hematopoiesisE. Increased erythrocyte number

30. A 35-year-old woman complains of heartpain (”aching and drilling”) occurring mai-nly in the morning in autumn and spring andirradiating to the neck, back and abdomen;rapid heartbeat; low vitality. Occurrence ofthis condition is not associated with physicalactivity. In the evening, the patient’s conditionimproves. Study of somatic and neurologicalstatus and ECG reveal no pathology. Whatpathology is most likely to have caused theseclinical presentations?

A. Somatization depressionB. Resting stenocardiaC. Pseudoneurotic schizophreniaD. Neurocirculatory astheniaE. Hypochondriacal depression

31. On the 4th day after recovery from acold a patient was hospitalized with complai-nts of solitary spittings of mucoid sputum.On the 2nd day there was a single dischargeof about 250 ml of purulent blood-streakedsputum. Objectively: the patient’s conditi-on is moderately severe. Respiratory rate -28-30/min., Ps- 96/min., BP- 110/70 mm Hg.Respiration over the left lung is vesicular,over the right lung - weakened. There arevarious moist crackles over the lower lobeand amphoric breath sounds near the angle ofscapula. What is the most likely diagnosis?

A. Acute pulmonary abscessB. Exudative pleuritisC. Acute focal pneumoniaD. Pleural empyemaE. Pyopneumothorax

32. A 65-year-old woman on abdominalpalpation presents with a tumor in the umbi-lical region and above it; the tumor is 13x8 cmin size, moderately painful, non-mobile, pulsi-ng. On auscultation systolic murmur can beobserved. What is the most likely diagnosis?

A. Abdominal aneurysmB. Gastric tumorC. Arteriovenous aneurysmD. Tricuspid insufficiencyE. Bicuspid insufficiency

Page 7: Department of human resources policy, education …D. Pelvioperitonitis E. Incipient abortion 5. 5 weeks after hypothermia a 22-year-old patient developed fever, weakness, muscle pain,

Крок 2 Medicine (англомовний варiант, iноземнi студенти) 2017 рiк 5

33. A 45-year-old man complains of cough fi-ts and tickling in his nasopharynx. He hadbeen staying for 10 days in the polluted areacreated by the Chornobyl nuclear power plantaccident. Rhinoscopy shows signs of severenasopharynx irritation. What radionuclide isthe cause of this irritation?

A. Radioactive iodineB. Radioactive cesiumC. Radioactive strontiumD. Radioactive plutoniumE. Radioactive cobalt

34. A 20-year-old patient complains of severeheadache, double vision, weakness, fever,irritability. Objectively: body temperature is38,1oC , the patient is reluctant to contact,sensitive to stimuli. There are ptosis ofthe left eyelid, exotropia, anisocoria S>D,pronounced meningeal syndrome. On lumbarpuncture the cerebrospinal fluid flowed outunder a pressure of 300 mm Hg, the fluidis clear, slightly opalescent. 24 hours laterthere appeared fibrin film. Protein - 1,4 g/l,lymphocytes - 600/3 per mm3, sugar - 0,3mmol/l. What is the provisional diagnosis?

A. Tuberculous meningitisB. Meningococcal meningitisC. Lymphocytic (Armstrong’s) meningitisD. Syphilitic meningitisE. Mumps meningitis

35. A 37-year-old worker during a fire endedup in the area of high CO concentration.He was delivered to a hospital in unconsci-ous state. Objectively: the skin of his faceand hands is crimson. Respiration rate is20/min. ECG: alterations specific for hypoxicmyocardium. Hourly diuresis is 40 ml. Bloodtest: erythrocytes - 4, 5 · 1012/l, Нb- 136g/l, color index - 0,89, ESR- 3 mm/hour,carboxyhemoglobin - 5%. What criterionallows determining the severity of the pati-ent’s condition?

A. Carboxyhemoglobin concentrationB. Respiratory disordersC. ECG resultsD. Extent of trophic disordersE. Development of chronic renal failure

36. After a case of purulent otitis a 1-year-oldboy has developed pain in the upper third ofthe left thigh, body temperature up to 39oC .Objectively: swelling of the thigh in its upperthird and smoothed out inguinal fold. The li-mb is in semiflexed position. Active and passi-ve movements are impossible due to severepain. What diagnosis is the most likely?

A. Acute hematogenous osteomyelitisB. Acute coxitisC. Intermuscular phlegmonD. OsteosarcomaE. Brodie’s abscess

37. A patient with suspected pheochromocytomahas normal blood pressure in the periodsbetween the atacks and a tendency towardstachycardia. Urine test revealed no pathologi-es. It was decided to use a provocative testwith histamine. What medication should beprepared to provide emergency care in caseof positive test result?

A. PhentolamineB. PipolphenC. NifedipineD. MesatonumE. Prednisolone

38. A 42-year-old woman complains of severepulsing headache in the frontoparietal area,vertigo, palpitations. She has been sufferi-ng from hypertension for 3 years. Significantincrease of BP occurs 2-3 times per monthand lasts for 3-8 hours. The left ventricle isenlarged, heart sounds are clear, heart rate- 105/min., BP- 225/115 mm Hg. ECG: signsof left ventricular hypertrophy. What drugwould be the most effective for terminationof cerebral crisis attack?

A. LabetalolB. Hydrochlorothiazide (Hypothiazide)C. CaptoprilD. LosartanE. Clonidine (Clophelin)

39. A 45-year-old patient complains of painin the epigastric region, left subcostal area,abdominal distension, diarrhea, loss of wei-ght. He has been suffering from this conditionfor 5 years. Objectively: the tongue is moistwith white coating near the root; deep palpati-on of abdomen reveals slight pain in the epi-gastric region and Мауо-Robson’s point. Li-ver is painless and protrudes by 1 cm from thecostal arch. Spleen cannot be palpated. Whatdisease can be primarily suspected?

A. Chronic pancreatitisB. Atrophic gastritisC. Peptic stomach ulcerD. Chronic cholecystitisE. Chronic enteritis

40. For three years a 31-year-old woman hasbeen complaining of pain and swelling ofher radiocarpal and metacarpophalangealarticulations, their reduced mobility in themorning, which persisted up to 1,5 hours.Two weeks ago she developed pain, swelli-ng, and reddening of her knee joints, herbody temperature increased up to 37,5oC .The treatment was belated. Examination ofthe internal organs revealed no pathologicalterations. Diagnosis of rheumatoid arthritiswas made. What alterations are the most li-kely to be visible on the arthrogram?

Page 8: Department of human resources policy, education …D. Pelvioperitonitis E. Incipient abortion 5. 5 weeks after hypothermia a 22-year-old patient developed fever, weakness, muscle pain,

Крок 2 Medicine (англомовний варiант, iноземнi студенти) 2017 рiк 6

A. Joint space narrowing, usurationB. Joint space narrowing, subchondralosteosclerosisC. Cysts in the subchondral boneD. Numerous marginal osteophytesE. Epiphyseal osteolysis

41. A 58-year-old woman complains ofspontaneous bruises, weakness, bleedinggums, dizziness. Objectively: the mucousmembranes and skin are pale with numeroushemorrhages of various time of origin. Lymphnodes are not enlarged. Ps- 100/min, BP-110/70 mm Hg. There are no alterations ofinternal organs. Blood test results: RBC-3, 0 · 1012/l, Нb- 92 g/l, color index - 0,9,anisocytosis, poikilocytosis, WBC- 10 · 109/l,eosinophils - 2%, stab neutrophils - 12%,segmented neutrophils - 68%, lymphocytes- 11%, monocytes - 7%, ESR- 12 mm/h. Whatlaboratory test should be performed additi-onally to make the diagnosis?

A. PlateletsB. ReticulocytesC. Clotting timeD. Osmotic resistance of erythrocytesE. Fibrinogen

42. After tonsillectomy a woman withsystemic lupus erythematosus, who has beentaking prednisolone for a year, developedacute weakness, nausea, vomiting, pain in theright iliac area, watery stool up to 5 timesper day. Ps- 96/min., BP- 80/50 mm Hg. Whatpreventive therapy should have been admini-stered prior to the surgery?

A. Increase of prednisolone dosageB. Prescription of desoxycorticosteroneacetateC. Prescription of antibioticsD. Administration of Ringer’s solutionE. Administration of 10% NaCl solution

43. A 45-year-old woman complains ofparoxysmal intolerable facial pain on the leftwith attacks that last for 1-2 minutes. Attacksare provoked by chewing. The disease onsetwas two month ago after overexposure tocold. Objectively: pain at the exit points of thetrigeminal nerve on the left. Touching nearthe wing of nose on the left induces new painattack with tonic spasm of the facial muscles.What is the most likely diagnosis among thoselisted?

A. Trigeminal neuralgiaB. Glossopharyngeal neuralgiaC. Temporomandibular joint arthritisD. Facial migraineE. Maxillary sinusitis

44. A 28-year-old man complains of skinrash and itching on the both of his hands.The condition persists for 1,5 years. Theexacerbation of his condition he ascribes tothe occupational contact with formaldehyde

resins. Objectively: lesion foci are symmetri-cally localized on both hands. Against thebackground of erythema with blurred margi-ns there are papulae, vesicles, erosions, crusts,and scales. What is the most likely pathology?

A. Occupational eczemaB. Idiopathic eczemaC. Allergic dermatitisD. Simple contact dermatitisE. Erythema multiforme

45. A 10-year-old boy with symptoms ofarthritis and myocarditis was delivered intoa hospital. Based on clinical examination thepreliminary diagnosis of juvenile rheumatoidarthritis was made. What symptom is the mostcontributive for the diagnostics of this di-sease?

A. Reduced mobility of the joints in themorningB. Regional hyperemia of the jointsC. Affection of the large jointsD. Enlarged heartE. Increased heart rate

46. A 25-year-old patient was delivered toan infectious diseases unit on the 3rd day ofillness with complaints of headache, pain inthe lumbar spine and gastrocnemius muscles,high fever, chills. Objectively: condition is ofmoderate severity. Scleras are icteric. Pharynxis hyperemic. Tongue is dry with dry browncoating. Abdomen is distended. Liver isenlarged by 2 cm. Spleen is not enlarged.Palpation of muscles, especially gastrocnemi-us muscles, is painful. Urine is dark in color.Feces are normal in color. What is the mostlikely diagnosis?

A. LeptospirosisB. Viral hepatitis type AC. MalariaD. Infectious mononucleosisE. Yersiniosis

47. A 28-year-old woman complains ofskin hemorrhages after minor traumas andspontaneous appearance of hemorrhages onthe front of her torso and extremities. Onexamination: the skin is variegated (old andnew hemorrhages), bleeding gums. Bloodplatelets - 20·109/l; in the bone marrow there isincreased number of megakaryocytes and noplatelet production. Treatment with steroidhormones was effective. What disease is it?

A. Idiopathic thrombocytopenic purpuraB. HemophiliaC. Rendu-Osler-Weber disease (Hereditaryhemorrhagic telangiectasia)D. Disseminated intravascular coagulationE. Acute vascular purpura

48. A 74-year-old patient was delivered intoadmission room with clinical presentations ofacute deep vein thrombosis of the shin. What

Page 9: Department of human resources policy, education …D. Pelvioperitonitis E. Incipient abortion 5. 5 weeks after hypothermia a 22-year-old patient developed fever, weakness, muscle pain,

Крок 2 Medicine (англомовний варiант, iноземнi студенти) 2017 рiк 7

symptom is the most typical of this pathology?

A. Homans’ signB. Rovsing’s signC. Courvoisier’s signD. Mayo-Robson’s signE. Grey Turner’s sign

49. A 9-month-old child presents with fever,cough, dyspnea. The symptoms appeared 5days ago after a contact with a person wi-th URTI. Objectively: the child is in gravecondition. Temperature is 38oC , cyanosis ofnasolabial triangle is present. RR- 54/min,nasal flaring during breathing is observed.There was percussion dullness on the rightbelow the scapula angle and tympanic soundover the other areas of lungs. Auscultati-on revealed bilateral fine moist cracklespredominating on the right. What is the mostlikely diagnosis?

A. PneumoniaB. URTIC. Acute laryngotracheitisD. Acute bronchitisE. Acute bronchiolitis

50. The mother of a 3-month-old child cameto a family doctor with complaints of herchild being physically underdeveloped andsuffering from cough attacks and dyspnea.Anamnesis: the child is the result of thesecond full-term pregnancy with the risk ofmiscarriage (the first child died of pulmonarypathology at the age of 4 months, accordingto the mother). Body mass at birth is 2500g. Cough attacks were observed from the fi-rst days of life, twice the child was treatedfor bronchitis. Considering the severity of thechild’s condition the doctor made the referralfor hospitalization. What diagnosis was mostlikely stated in the referral?

A. Mucoviscidosis (Cystic fibrosis)B. Acute obstructive bronchitisC. Recurrent obstructive bronchitisD. PertussisE. Acute obstructive pneumonia

51. A 46-year-old man notes swollen legs,weakness, sensation of fullness and heavi-ness in the right subcostal area; it is thefirst occurrence of these signs in the pati-ent. The patient has 20-year-long history ofrheumatoid arthritis. The liver and spleenare enlarged and dense. Blood creatinine -0,23 mmol/l, proteinemia - 68 g/l, cholesterol- 4,2 mmol/l, urine specific gravity - 1012,proteinuria - 3,3 g/l, isolated wax-like cyli-nders, leached erythrocytes in the vision field,leukocytes - 5-6 in the vision field. What is themost likely complication?

A. Renal amyloidosisB. Chronic glomerulonephritisC. Acute glomerulonephritisD. Heart failureE. Chronic pyelonephritis

52. A 23-year-old man had taken 1 g of aspirinto treat acute respiratory infection. After thathe developed an asthmatic fit with laboredexpiration that was arrested by introductionof aminophylline. The patient has no medicalhistory of allergies. The patient has undergonetwo surgeries for nasal polyposis in the past.What is the most likely diagnosis?

A. Aspirin-induced asthmaB. Atopic bronchial asthmaC. Infectious allergic bronchial asthmaD. Exercise-induced asthmaE. Symptomatic bronchospasm

53. A 50-year-old patient was delivered toa hospital with complaints of blood tracesin urine. Urination is painless and undi-sturbed. Macrohematuria had been observedfor 3 days. Objectively: kidneys cannot bepalpated, suprapubic area is without alterati-ons, external genitalia are non-pathologic. Onrectal investigation: prostate is not enlarged,painless, has normal structure. Cystoscopyrevealed no alterations. What is the most li-kely diagnosis?

A. Renal carcinomaB. Bladder tuberculosisC. VaricoceleD. Dystopic kidneyE. Necrotic papillitis

54. A man complains of constant dull painin the perineum and suprapubic area, weakflow of urine, frequent difficult painfulurination, nocturia. The patient has beensuffering from this condition for severalmonths, during which urination was becomi-ng increasingly difficult, and pain in the peri-neum has developed. On rectal examinati-on: the prostate is enlarged (mainly its rightlobe), dense, asymmetrical, central fissure issmoothed out, the right lobe is of stony densi-ty, painless, tuberous. What disease is it?

A. Prostate cancerB. Prostate sclerosisC. Urolithiasis, prostatolith of the right lobeD. Prostate tuberculosisE. Chronic congestion prostatitis

55. A boy was born at 32 weeks of gestation. 2hours after the birth he developed respiratorydistress (RD). The RD severity assessed bySilverman score was 5. The respiratory di-sorders progressed, respiratory failure couldnot be eliminated by Martin-Bouyer CPAP(continuous positive airway pressure). X-rayof lungs shows reticular and nodular pattern,air bronchogram. What is the most likelycause of respiratory distress syndrome?

Page 10: Department of human resources policy, education …D. Pelvioperitonitis E. Incipient abortion 5. 5 weeks after hypothermia a 22-year-old patient developed fever, weakness, muscle pain,

Крок 2 Medicine (англомовний варiант, iноземнi студенти) 2017 рiк 8

A. Hyaline membrane diseaseB. Segmental atelectasisC. Bronchopulmonary dysplasiaD. Congenital pulmonary emphysemaE. Edematous hemorrhagic syndrome

56. An 8-year-old child with a 3-year-longhistory of diabetes was hospitalized inhyperglycemic coma. Specify the initial doseof insulin to be administered:

A. 0,1-0,2 U/kg of body weight per hourB. 0,05 U/kg of body weight per hourC. 0,2-0,3 U/kg of body weight per hourD. 0,3-0,4 U/kg of body weight per hourE. 0,4-0,5 U/kg of body weight per hour

57. A 17-year-old young man complains ofgeneral weakness, trismus, twitching of themuscles in his left shin. 7 days ago he piercedhis foot with a nail. Objectively: at the soleof the foot there is a wound, 0,3х0,2 mm insize, with small amount of serous-purulentdischarge, the skin around the wound ishyperemic. What is the most likely diagnosis?

A. TetanusB. PhlegmonC. OsteomyelitisD. Infected woundE. Erysipelas

58. A patient with trauma of the lower thi-rd of the forearm volar surface caused by aglass shard came to a first-aid center. Objecti-vely: flexion of the IV and V fingers isimpaired, sensitivity of the inner dorsal andpalmar surfaces of the hand and IV finger isdecreased. What nerve is damaged?

A. UlnarB. RadialC. MedianD. MusculocutaneousE. Axillary

59. A 58-year-old patient complains ofpain in the lower left extremity, whichaggravates during walking, sensation of coldand numbness in the both feet. The patienthas been suffering from this condition for 6years. Objectively: the skin is pale and dry,with hyperkeratosis. On the left shin hair isscarce, ”furrow” symptom of inflamed ingui-nal lymph nodes is positive. Pulse cannot bedetected over the pedal and popliteal arteri-es and is weakened over the femoral artery.In the right limb popliteal artery pulsation isretained. What is the most likely diagnosis?

A. Arteriosclerosis obliterans of the lowerextremitiesB. Obliterating endarteritisC. Femoral artery thrombosisD. Raynaud’s diseaseE. Buerger’s disease (thromboangiitis obli-terans)

60. An infant is 2,5 months old. The onset ofthe disease was gradual, the child had normalbody temperature but presented with slightcough. Within a week the cough intensified,especially at night; on the 12th day the childdeveloped cough fits occurring up to 20 timesper day and followed by vomiting. There wasone instance of respiratory arrest. Make thediagnosis:

A. PertussisB. ParainfluenzaC. Congenital stridorD. Respiratory syncytial infectionE. Adenovirus infection

61. Vaginal examination reveals the head ofthe fetus, which fills the posterior surface ofsymphysis pubis and hollow of the sacrum.The lower edge of symphysis pubis, ischi-adic spines, and sacrococcygeal joint can bepalpated. Where in the lesser pelvis is the fetalhead situated?

A. In the narrow plane of lesser pelvisB. In the wide plane of lesser pelvisC. Above pelvic inletD. In the area of brimE. In the area of pelvic outlet

62. A man works in casting of nonferrousmetals and alloys for 12 years. In the airof working area there was registered highcontent of heavy metals, carbon monoxi-de, and nitrogen. During periodic healthexamination the patient presents withasthenovegetative syndrome, sharp painsin the stomach, constipations, pain in thehepatic area. In urine: aminolevulinic acidand coproporphyrin are detected. In blood:reticulocytosis, low hemoglobin level. Suchintoxication is caused by:

A. Lead and lead saltsB. TinC. Carbon monoxideD. Nitric oxideE. Zinc

63. A 30-year-old multigravida has beenin labour for 18 hours. 2 hours ago thepushing stage began. Fetal heart rate isclear, rhythmic, 136/min. Vaginal examinati-on reveals complete cervical dilatation, thefetal head in the pelvic outlet plane. Sagittalsuture is in line with obstetric conjugate, theoccipital fontanel is near the pubis. The pati-ent has been diagnosed with primary uterineinertia. What is the further tactics of labourmanagement?

A. Outlet forcepsB. Labour stimulationC. Cesarean sectionD. Skin-head Ivanov’s forcepsE. Vacuum extraction of the fetus

64. A patient, who had eaten canned

Page 11: Department of human resources policy, education …D. Pelvioperitonitis E. Incipient abortion 5. 5 weeks after hypothermia a 22-year-old patient developed fever, weakness, muscle pain,

Крок 2 Medicine (англомовний варiант, iноземнi студенти) 2017 рiк 9

mushrooms (honey agaric) three daysago, developed vision impairment (di-plopia, mydriasis), speech disorder, disturbedswallowing. What type of food poisoningoccurred in the patient?

A. BotulismB. Food toxicoinfectionC. FusariotoxicosisD. Honey agaric poisonongE. Lead salts poisoning

65. An infant is 3 weeks old. Since birththere has been observed periodical vomiti-ng within a few minutes after feeding. Theamount of vomitive masses does not exceedthe volume of previous feeding. The infanthas age-appropriate body weight. What is themost likely cause of this symptom?

A. PylorospasmB. Esophageal chalasiaC. Adrenogenital syndromeD. PyloristenosisE. Esophageal achalasia

66. A newborn with gestational age of 31weeks presents with hypotonia and depressedconsciousness. Hematocrit is 35%, generalcerebrospinal fluid analysis shows increasedcontent of erythrocytes and protein, and lowglucose. These data correspond with the clini-cal presentation of:

A. Intracranial hemorrhageB. MeningitisC. SepsisD. AnemiaE. Prenatal infection

67. From urine of a 14-year-old boy withthe exacerbation of secondary obstructivepyelonephritis Pseudomonas aeruginosa wasisolated with a titer of 1000000 microbes per1 ml. What antibiotic is the most advisable inthis case?

A. CiprofloxacinB. AmpicillinC. CefazolinumD. AzithromycinE. Chloramphenicol

68. A 20-year-old woman on the 10th dayafter her discharge from the maternity warddeveloped fever up to 39oC and pain in herleft mammary gland. On examination themammary gland is enlarged, in its upper outerquadrant there is a hyperemic area. In thisarea a dense spot with blurred margins can bepalpated. The patient presents with lactostasisand no fluctuation. Lymph nodes in the ri-ght axillary crease are enlarged and painful.Specify the correct diagnosis:

A. Lactational mastitisB. AbscessC. ErysipelasD. DermatitisE. Tumor

69. A 3-year-old girl is being treated at aresuscitation unit with diagnosis ”acute ki-dney failure, oligoanuric stage”. ECG: high Twave, extended QRS complex, displacementof S-T interval downwards below the isoline.What electrolyte imbalance is it?

A. HyperkalemiaB. HypokalemiaC. HypocalcemiaD. HypercalcemiaE. Hyperphosphatemia

70. Mother of an 8-year-old girl complainsthat the child is too short and has excessi-ve body weight. Objectively: obesity with fatdeposits on the torso and face (round moon-like face), acne, striae on the thighs and lowerabdomen, hirsutism. What hormone can causesuch symptoms, when in excess?

A. CortisolB. ThyroxinC. TestosteroneD. InsulinE. Glucagon

71. A 9-year-old girl complains of fever up to37,5oC , headache, inertness, weakness, lossof appetite, stomachache, and frequent pai-nful urination. Provisional diagnosis of acutepyelonephritis is made. Clinical urine analysis:specific gravity - 1018, no protein, leukocytes- 10-15 in the vision field. What investigati-on method can verify the diagnosis of urinarysystem infection?

A. Bacteriological inoculation of urineB. Rehberg test (creatinine clearance test)C. Zymnytsky test (measurement of dailydiuresis)D. Complete blood countE. Clinical urine analyses, dynamic testing

72. During assessment of work conditionsat the mercury thermometer manufacture,content of mercury vapors in the air of worki-ng area is revealed to exceed maximumconcentration limit. Specify the main way ofmercury penetration into the body:

A. Respiratory organsB. Intact skinC. Damaged skinD. Gastrointestinal tractE. Mucous tunics

73. During health assessment of car driversand police officers on point duty, the physici-ans detected carboxyhemoglobin in the bloodof the patients, weakened reflex responses,disturbed activity of a number of enzymes.

Page 12: Department of human resources policy, education …D. Pelvioperitonitis E. Incipient abortion 5. 5 weeks after hypothermia a 22-year-old patient developed fever, weakness, muscle pain,

Крок 2 Medicine (англомовний варiант, iноземнi студенти) 2017 рiк 10

Revealed professional health disorders aremost likely to be associated with the effect of:

A. Carbon monoxideB. Sulfurous anhydrideC. Mental stressD. Aromatic hydrocarbonsE. Nitric oxide

74. On the 3rd day of life a newborn,who had suffered birth asphyxia, developedhemorrhage from the umbilical wound.Laboratory analysis reveals hypocoagulati-on, thrombocytopenia, and hypothrombi-nemia. What is the cause of such clinicaldevelopments?

A. Disseminated intravascular coagulationB. Hemorrhagic disease of newbornC. Congenital angiopathyD. Thrombocytopenic purpuraE. Umbilical vessel trauma

75. A 46-year-old woman came to a materni-ty clinic with complaints of moderate blooddischarge from the vagina, which developedafter the menstruation delay of 1,5 months.On vaginal examination: the cervix is clean;the uterus is not enlarged, mobile, painless;appendages without changes. Make the di-agnosis:

A. Dysfunctional uterine bleedingB. AdenomyosisC. Ectopic pregnancyD. Submucous uterine myomaE. Cancer of the uterine body

76. A patient has the second and third degreeburns of the 15% of the body surface. On the20th day after the trauma the patient presentswith sharp increase of body temperature,general weakness, rapid vesicular respiration;facial features are sharpened, BP is 90/50 mmHg, heart rate is 112/min. What complicationis it?

A. SepsisB. PneumoniaC. Acute intoxicationD. Purulent bronchitisE. Anaerobic infection

77. A 30-year-old woman complains ofamenorrhea that lasts for 2 years after shehas given birth, loss of hair and body weight.The labor was complicated with hemorrhagecaused by uterine hypotonia. Objectively thepatient is of asthenic type, her external geni-talia are hypoplastic, the uterine body is smallin size and painless. No uterine appendagescan be detected. What is the most likely di-agnosis?

A. Sheehan’s syndrome (postpartum hypopi-tuitarism)B. Ovarian amenorrheaC. Turner’s syndromeD. Ovarian exhaustion syndromeE. Galactorrhea-amenorrhea syndrome

78. On laboratory investigation of a porksample there is 1 dead trichinella detectedin 24 sections. This meat should be:

A. Handed over for technical disposalB. Allowed for sale with no restrictionsC. Processed and sold through public cateringnetworkD. Processed for boiled sausage productionE. Frozen until the temperature of - 10oC isreached in the deep layers, with subsequentexposure to cold for 15 days

79. Examination of a 43-year-old manobjectively revealed pallor of skin andmucous membranes, loss of tongue papillae,transverse striation of fingernails, cracks inthe mouth corners, tachycardia. Blood testresults: Hb- 90 g/l, anisocytosis, poikilocytosis.The most likely causative factor of this condi-tion is the inadequate intake of:

A. IronB. CopperC. ZincD. MagnesiumE. Selene

80. A 52-year-old woman presents withaffected mucosa in the mouth angles wherefissures, erosions, and ulcers develop; verti-cal fissures appear on the lips during theirclosing (cheilosis); there are tongue alterati-ons (glossitis), angular stomatitis, seborrheicdermatitis around the mouth and wings ofthe nose, and pericorneal injection. The listedsymptoms are characteristic of:

A. B2-hypovitaminosisB. B1-hypovitaminosisC. C-hypovitaminosisD. PP -hypovitaminosisE. A-hypovitaminosis

81. Two days ago a woman fell from the hei-ght of 1,5 m. She complains of severe thoracicpain on the left and dyspnea. Chest X-rayreveals hydropneumothorax on the left withfluid level at the 7th rib and the lung collapsedby 1/3. The 6th-7th ribs are fractured alongthe scapular line. Serohemorrhagic fluid wasobtained during thoracic puncture. Whattreatment tactics should be chosen?

Page 13: Department of human resources policy, education …D. Pelvioperitonitis E. Incipient abortion 5. 5 weeks after hypothermia a 22-year-old patient developed fever, weakness, muscle pain,

Крок 2 Medicine (англомовний варiант, iноземнi студенти) 2017 рiк 11

A. Thoracocentesis on the left at the 7thintercostal spaceB. Thoracocentesis at the 2nd intercostal spaceC. Repeated puncture of the pleural cavityD. Intubation of trachea with artificialpulmonary ventilationE. Alcohol-novocaine block of the fracturearea

82. A patient complains of suppuration fromthe ear and impaired hearing of the left ear,which have been observed for the past 6years. The patient had periodical headaches,general indisposition, fever. Objectively:otoscopy of the external auditory meatusrevealed mucopurulent odorless substance.The eardrum is of normal color, with centralperforation. What is the most likely di-agnosis?

A. Chronic mesotympanitisB. OtosclerosisC. Acute otitis mediaD. Chronic epitympanitisE. Chronic sensorineural hearing loss

83. In 2 hours after a traffic accident a 28-year-old man in grave condition was delivered to ahospital. The patient complains of abdominalpain. He received a blow to the abdomen wi-th the steering wheel. Objective examinationrevealed the following: the abdomen does notparticipate in respiration, is tense and acutelypainful on palpation; the abdominal musclesare defensively tense, peritoneal irritation si-gns are positive, hepatic dullness is absent. BPis 90/60 mm Hg, heart rate is 120/min. Whatfurther treatment tactics should be chosen?

A. LaparotomyB. LaparoscopyC. Cold to the abdomenD. Ultrasound investigationE. Laparocentesis

84. A 28-year-old woman complains ofincreased intervals between menstruations,up to 2 months, and hirsutism. Gynecologicalexamination revealed the following: ovariesare enlarged, painless, and dense; no alterati-ons of the uterus. US of the lesser pelvis:ovaries are 4-5 cm in diameter, with numerousenlarged follicles on the periphery. X-ray ofthe skull base: sellar region is widened. Whatis the most likely diagnosis?

A. Stein-Leventhal syndrome (polycysticovarian syndrome)B. AlgodismenorrheaC. Sheehan syndrome (postpartum hypopitui-tarism)D. Premenstrual syndromeE. Morgagni-Stewart-Morel syndrome(metabolic craniopathy)

85. Posture of an 11-year-old boy was determi-ned during preventive examination. The chi-ld presents with curled forward rounded

shoulders, the head is bowed forward, thethorax is flattened, the stomach is bulging.In the vertebral column there are deepenedcervical and lumbar flexures. What posturedoes the child have?

A. KyphosisB. LordosisC. StoopingD. CorrectedE. Normal

86. A 46-year-old woman has been hospitali-zed with open fracture of the left thigh inits middle third. She underwent the surgery- fixation with extraosseous osteosynthesisplates. On the 4th day after the surgeryshe developed pain in the wound, bodytemperature rose over 39oC . What measuresshould be taken in this case?

A. Undo the sutures, drain the wound, andprescribe antibioticsB. Prescribe broad spectrum antibiotics andhormonal agentsC. Administer antibiotics intraosseously andhypothermia locallyD. Inject antibiotics into the area surroundi-ng the wound, prescribe spasmolytics andanalgesicsE. Remove the fixation, prescribe sulfanilami-des

87. A multigravida on the 38th week of herpregnancy complains of increased BP up to140/90 mm Hg, edema of the shins for 2 weeks.In the last month she gained 3,5 kg of weight.Urine analysis: protein - 0,033 g/l. Make thediagnosis:

A. Mild preeclampsiaB. Moderate preeclampsiaC. Pregnancy hypertensionD. Severe preeclampsiaE. Pregnancy edema

88. A 44-year-old patient with postinfarcti-on cardiosclerosis presents with frequentheart rate disorders and lower extremityedema. Objectively: Ps- 95/min., irregular,10-12 extrasystoles per minute. BP- 135/90mm Hg. The 1st heart sound at the apex isweakened. Pulmonary respiration is rough.The liver is enlarged +2 cm. ECG: irregularsinus rhythm, heart rate - 95/min, frequentpolytopic ventricular extrasystoles. What anti-arrhythmic drug is advisable in this case fortreatment and prevention of extrasystole?

A. AmiodaroneB. LidocaineC. MexiletineD. QuinidineE. Novocainamide (Procainamide)

89. A 60 year-old woman has been sufferingfrom weakness, dizziness, and fatigue overthe last year. Recently she has also developed

Page 14: Department of human resources policy, education …D. Pelvioperitonitis E. Incipient abortion 5. 5 weeks after hypothermia a 22-year-old patient developed fever, weakness, muscle pain,

Крок 2 Medicine (англомовний варiант, iноземнi студенти) 2017 рiк 12

dyspnea, paresthesia. Objectively: the skinand mucous membranes are pale and slightlyicteric. The tongue is smooth due to the loss oflingual papillae. Liver and spleen are locatedat the costal margin. Blood count: Hb- 70 g/l,RBC- 1, 7·1012/l, color index - 1,2, macrocytes.Administer the patient a pathogeneticallyjustified drug:

A. Vitamin B12B. Vitamin B6C. Ascorbic acidD. Iron preparationsE. Vitamin B1

90. After excessive consumption of fatty fooda 60-year-old woman suddenly developedpain in her right subcostal area, nausea, bi-le vomiting, sharp bitter taste in her mouth.In 2 days she developed jaundice, her uri-ne darkened. Objectively: sclera and skin areicteric, the abdomen is distended, the liver isenlarged by 3 cm, soft and painful on palpati-on, Ortner’s, Murphy’s, Kehr’s, Zakharyin’s,Mayo-Robson’s signs are positive. What di-agnostic technique should be used in the firstplace?

A. US of the gallbladder and bile ductB. FibrogastroduodenoscopyC. X-ray of the abdomenD. Radionuclide scanning of the liver andgallbladderE. Laparoscopy

91. A 42-year-old woman working at a poultryfarm complains of dyspnea, thoracic pain onthe left, increased body temperature up to 38-39oC in the evening, and cough. The patientsuffers from essential hypertension. Objecti-vely: vesicular respiration in the lungs, vesi-cular resonance without alterations. X-rayof both lungs reveals numerous small low-intensity foci, 2-3 mm in size, located in a rowalong the blood vessels. ESR- 32 mm/hour.What is the most likely diagnosis?

A. Acute disseminated tuberculosisB. Infiltrative tuberculosisC. Pulmonary carcinomatosisD. Focal tuberculosisE. Community-acquired pneumonia

92. A 65-year-old patient has been sufferingfrom liver cirrhosis associated with hepati-tis C virus for 7 years. During the last 3weeks the patient developed severe edemaof the lower extremities, the abdomen is si-gnificantly distended with fluid. Ultrasound:signs of liver cirrhosis, portal hypertensi-on, lymph nodes are in the area of hepaticportal. Concentration of α-fetoprotein inblood serum is 285 ng/ml. What complicati-on could have developed in this case?

A. Hepatocellular carcinoma of the liverB. Hepatocellular insufficiencyC. HypersplenismD. CholestasisE. Congestive heart failure

93. A 43-year-old man has undergone asurgery for osteomyelitis of the left thigh.On the 6th day the patient’s condition wascomplicated with sepsis. Despite complextherapy of sepsis on the 9th day the patientpersistently presents with temperature up to40oC , heart rate is 110/min., respiration rateis 23/min., BP is 100/60 mm Hg. Blood test:leukocytes - 16 ·109/l, band neutrophils - 16%.What phase of clinical course is it?

A. CatabolicB. AnabolicC. RehabilitationD. FunctionalE. Tension

94. A 23-year-old woman presents with di-abetes mellitus type 1. She complains ofweakness, headache, nausea, and vomiting.Objectively: temperature is 37,6oC , heart rateis 98/min., BP is 95/65 mm Hg, respiration rateis 32/min., loud. Smell of acetone is detected,heart sounds are muffled, pulse is rhythmic.The stomach is sensitive in the epigastrium.Costovertebral angle tenderness (Murphy’spunch sign) is present. Blood glucose is 28,5mmol/l; blood leukocytes - 16, 5 · 109/l. In uri-ne: acetone ++, leukocytes - 25-40 in the visionfield. Blood pH is 7,1. What correction methodwould be the most advisable?

A. Infusion of normal saline + insulinotherapyin small portionsB. Infusion of 5% glucose + insulinotherapy inlarge portionsC. Infusion of 2,5% sodium bicarbonate +insulinotherapyD. Intravenous administration of antibiotics +insulinotherapyE. Infusion of dextran solutions + insuli-notherapy

95. A 57-year-old woman complains of asensation of esophageal compression, palpi-tations, difficult breathing when eating solidfood, occasional vomiting with a full mouth,”wet pillow” sign at night for the last 6months. Objectively: body tempearture -39oC , height - 168 cm, weight - 72 kg, Ps-76/min, BP- 120/80 mm Hg. X-ray studyrevealed considerable dilation of esophagusand its constriction in the cardial part. Whatpathology is the most likely to have causeddysphagia in this patient?

A. Achalasia cardiaeB. Primary esophageal spasmC. Hiatal herniaD. Esophageal carcinomaE. Reflux esophagitis

Page 15: Department of human resources policy, education …D. Pelvioperitonitis E. Incipient abortion 5. 5 weeks after hypothermia a 22-year-old patient developed fever, weakness, muscle pain,

Крок 2 Medicine (англомовний варiант, iноземнi студенти) 2017 рiк 13

96. A 47-year-old man is employed at theweaving workshop, has 15-year-long recordof service at this factory; his work conditi-ons are associated with high-frequency andhigh-intensity noise. During periodical exami-nation he was diagnosed with occupationaldeafness. What are the grounds for makingsuch a diagnosis?

A. Audiometry data and hygienic assessmentof working environmentB. Record of service at this factoryC. Noise characteristic at this factoryD. Central nervous system examination resultsE. Inner ear examination results

97. A 37-year-old woman complains of acutepain in the genital area, swelling of thelabia, pain when walking. Objectively: bodytemperature is 38,7oC , Ps- 98/min. In theinterior of the right labia there is a dense,painful tumor-like formation 5,0x4,5 cm in si-ze, the skin and mucous membrane of genitalsare hyperemic, there is profuse foul-smellingdischarge. What is the most likely diagnosis?

A. Acute bartholinitisB. Labial furuncleC. Acute vulvovaginitisD. Bartholin gland cystE. Carcinoma of vulva

98. A woman undergoing in-patient treatmentfor viral hepatitis type B developed headache,nausea, recurrent vomiting, memory lapses,flapping tremor of her hands, rapid pulse.Sweet smell from the mouth is detected. Bodytemperature is 37,6oC , heart rate is 89/min.What complication developed in the patient?

A. Acute liver failureB. Ischemic strokeC. Gastrointestinal hemorrhageD. Hypoglycemic shockE. Meningoencephalitis

99. A 42-year-old man was delivered to asurgical in-patient department with complai-nts of icteric skin, pain in the right subcostalarea. Biochemical blood analysis: total bili-rubin - 140 mcmol/l, direct bilirubin - 112mcmol/l. On US: choledoch duct - 1,4 cm,a concrement is detected in the distal area.Gallbladder is 40 cm, no concrements. Whattreatment tactics should be chosen?

A. Endoscopic papillosphincterotomyB. Laparoscopic cholecystectomyC. Laparotomy with choledoch duct drainD. Laparotomy with cholecystectomyE. Threatment in an infectious diseases hospi-tal

100. 4 weeks after myocardial infarction a 56-year-old patient developed acute heart pain,marked dyspnea. Objectively: the patient’scondition is extremely grave, there is markedcyanosis of face, swelling and throbbing of

neck veins, peripheral pulse is absent, thecarotid artery pulse is rhythmic, 130/min., BPis 60/20 mm Hg. Auscultation of the heartreveals extremely muffled sounds, percussionreveals heart border extension in both directi-ons. What is the optimal treatment tactics forthis patient?

A. Pericardiocentesis and immediatethoracotomyB. Oxygen inhalationC. Puncture of the pleural cavity on the leftD. Conservative treatment, infusion ofadrenomimeticsE. Pleural cavity drainage

101. A patient with chronic pancreatitiscomplains of diarrhea occurring up to 5 ti-mes per day (no blood traces), loss of bodyweight, abdominal distention, dryness of skin,loss of hair, thirst, bleeding gums, convulsi-ons. Complete blood count: leukocytes -5, 8 · 109/l; Hb- 86 g/l; ESR- 15 mm/g; Bloodprotein test: protein - 48 g/l; albumins - 28g/l. What indicators of coprological analysiswould accompany this syndrom?

A. Steatorrhea, creatorrheaB. Large amount of mucus, amylorrheaC. Large amount of starch grains and celluloseD. Gas bubbles, acid reactionE. Large numbers of iodinophilous microbes

102. A 35-year-old patient complains ofheartburn, gasseous and sour eructation,burning constricting pain behind the sternumand along the esophagus, developing whenbowing the torso to the front. No previ-ous examination; the patient takes almagelat his own discretion, after which he notesimprovement of his general state. Make theprovisional diagnosis:

A. Gastroesophageal reflux diseaseB. Functional dyspepsiaC. CardiospasmD. Ulcer disease of the stomachE. Ulcer disease of the duodenum

103. The left hand of a newborn is extendedin all its joints, stretched along the torso, andpronated in the forearm. Active movementsof the shoulder joint are retained. The handis flattened, atrophied, cold to touch, hangspassively. Grasping and Babkin’s reflexes areabsent at the affected side. Hemogram indi-cators are normal. Make the most likely di-agnosis:

A. Inferior distal obstetrical paralysisB. OsteomyelitisC. Proximal obstetrical paralysisD. Complete obstetrical paralysisE. Hypoxic-ischemic encephalopathy

104. A 34-year-old man is being treated forschizophrenia exacerbation in a psychiatricunit. Objectively: the patient remains in bed,

Page 16: Department of human resources policy, education …D. Pelvioperitonitis E. Incipient abortion 5. 5 weeks after hypothermia a 22-year-old patient developed fever, weakness, muscle pain,

Крок 2 Medicine (англомовний варiант, iноземнi студенти) 2017 рiк 14

is sluggishly mobile, unresponsive, does notreact to questions. His position is unvari-ed, hypomimic, snout reflex and Dupre’ssymptom are present, muscles exhibit waxyflexibility. He has been remaining in thisstate for approximately a week. Feeding isparenteral. Determine the neuromotor di-sturbance:

A. Catatonic stuporB. Depressive stuporC. Psychogenic stuporD. Anergic stuporE. Exogenic stupor

105. A 14-year-old girl has been delivered toa gynecological department with complaintsof profuse blood discharge from her genitaltract for 2 weeks. Anamnesis: menstruation si-nce 13, irregular, painful, profuse; the last onewas 2 months ago. Objectively: pale skin andmucosa, BP- 100/60 mm Hg, Hb- 108 g/l. Theabdomen is soft and painless on palpation.Rectal examination revealed no pathologiesof reproductive organs. What condition is it?

A. Juvenile uterine hemorrhage (Dysfuncti-onal)B. Hypomenstrual syndromeC. Inflammation of uterine appendages (Pelvicinflammatory disease)D. PelviperitonitisE. Endometritis

106. An infant cries during urination, theforeskin swells and urine is excreted indrops. What approach to treatment shouldbe chosen?

A. Create an opening into the foreskin cavityB. Prescription of α-adrenergic blockingagentsC. Prescription of antispasmodic agentsD. Urinary bladder catheterizationE. Epicystostomy

107. A 20-year-old patient complains of painin the left lumbar region, arterial pressure riseup to 160/110 mm Hg. US revealed that thestructure and size of the right kidney were wi-thin age norms, there were signs of the 3rddegree hydronephrotic transformation of theleft kidney. Doppler examination revealedan additional artery running to the lowerpole of the kidney. Excretory urogram showsa narrowing in the region of ureteropelvicjunction. Specify the treatment tactics:

A. Surgical interventionB. Administration of spasmolyticsC. Administration of ACE inhibitorsD. Kidney catheterizationE. Administration of β-blockers

108. A 49-year-old patient consulted a doctorabout difficult swallowing, voice hoarseness,weight loss. These symptoms have beengradually progressing for the last 3 months.

Objectively: the patient is exhausted, thereare enlarged supraclavicular lymph nodes.Esophagoscopy revealed no esophagealpathology. Which of the following investigati-ons is the most appropriate in this case?

A. Computed tomography of chest and medi-astinumB. X-ray of lungsC. Multiplanar imaging of esophagusD. Radioisotope investigation of chestE. Ultrasound investigation of mediastinum

109. A 15-year-old girl complains of dizzi-ness and sensation of lack of air that shedevelops in emotionally straining situations.Relief occurs after she takes corvalol. Objecti-vely: hyperhidrosis and marble-like patternof the skin of her palms and feet. Clinicaland instrumental examination revealed noorganic alterations of the central nervous,cardiovascular, and respiratory systems. Whatprovisional diagnosis can be made?

A. Somatoform autonomic dysfunctionB. Obstructive bronchitisC. Bronchial asthmaD. Stenosing laryngotracheitisE. Acute epiglottitis

110. A 14-year-old patient complains ofalopecia foci on his scalp. The patient hasbeen presenting with this condition for 2weeks. Objectively: on the scalp there areseveral small oval foci with blurred margins.The skin in the foci is pink-red, the hairs arebroken off at 4-5 mm length or at skin level.Under Wood’s lamp there are no foci of greenluminescence detected. What disease is it?

A. Trichophytosis capitisB. Syphilitic alopeciaC. Alopecia areataD. SclerodermaE. Psoriasis

111. A 38-year-old patient has been deliveredby an ambulance to a surgical departmentwith complaints of general weakness, indi-sposition, black stool. On examination thepatient is pale, there are dotted hemorrhageson the skin of his torso and extremiti-es. On digital investigation there are blackfeces on the glove. Blood test: Hb- 108 g/l,thrombocytopenia. Anamnesis states that si-milar condition was observed 1 year ago.Make the diagnosis:

A. Thrombocytopenic purpuraB. HemophiliaC. Ulcerative bleedingD. Rectal tumorE. Nonspecific ulcerative colitis

112. A 35-year-old patient developed an epi-leptic attack with tonoclonic spasms thatlasted for 3 minutes. After the attack the pati-ent fell asleep but in 5 minutes the second

Page 17: Department of human resources policy, education …D. Pelvioperitonitis E. Incipient abortion 5. 5 weeks after hypothermia a 22-year-old patient developed fever, weakness, muscle pain,

Крок 2 Medicine (англомовний варiант, iноземнi студенти) 2017 рiк 15

attack occurred. The first step of emergencyaid would be to:

A. Ensure patency of airwaysB. Take blood from the vein for analysisC. Introduce diazepam intravenouslyD. Prescribe antiepileptic drugsE. Administer chloral hydrate via an enema

113. A 23-year-old woman has been sufferingfrom a mental disease since the age of 18,the course of disease has no remission peri-ods. At a hospital the patient mostly presentswith non-purposeful foolish excitation: shemakes stereotypic grimaces, exposes herself,publicly masturbates with a loud laughter,repeates stereotypical abusive shouts. Thepatient should be prescribed:

A. NeurolepticsB. AntidepressantsC. TranquilizersD. NootropicsE. Mood stabilizers

114. A young woman suffering fromseborrhea oleosa has numerous light-brownand white spots on the skin of her torsoand shoulders. The spots have clear margi-ns, branny desquamation, no itching. Whatprovisional diagnosis can be made?

A. Pityriasis versicolorB. Torso dermatophytosisC. Seborrheic dermatitisD. Pityriasis roseaE. Vitiligo

115. 10 days after birth a newborn developedsudden fever up to 38,1oC . Objectively: theskin in the region of navel, abdomen andchest is erythematous; there are multiple pea-sized blisters with no infiltration at the base;isolated bright red moist erosions with epi-dermal fragments are observed on the peri-phery. What is the provisional diagnosis?

A. Epidemic pemphigus of newbornB. Syphilitic pemphigusC. Streptococcal impetigoD. Vulgar impetigoE. Atopic dermatitis

116. A man came to an urologist withcomplains of painful urination, dischargefrom urethra. The patient has been sufferingfrom this condition for a week. Objectively:hyperemic urinary meatus, edema, purulentdischarge. Microscopy of smears detectedgram-negative bacteria. Specify the diagnosis:

A. Acute gonorrheal urethritisB. Trichomonas urethritisC. Candidal urethritisD. Chlamydial urethritisE. Chronic gonorrhea

117. A patient is 28 years old. He has beensuffering from mental disorder since he was

22. His current condition has changed acutely:for 3 days the patient has been refusingto leave his home. He claims that there isa ”telepathy” occurring between him andthe other people, through which he receives”thoughts of strangers” and transmits his ownthoughts for everyone to hear. He is convi-nced that his thoughts and actions are beingmanipulated through this ”telepathy”. Makethe preliminary diagnosis:

A. Paranoid schizophreniaB. Depressive episodeC. Manic episodeD. Organic deliriumE. Acute reaction to stress

118. A 35-year-old woman has gained 20 kgweight within a year with the normal diet.She complains of chills, sleepiness, shortnessof breath. The patient’s mother and sister arecorpulent. Objectively: height - 160 cm, wei-ght - 92 kg, BMI- 35,9. Obesity is uniform,there are no striae. The face is amimic. Theskin is dry. The tongue is thickened. Heartsounds are muffled. HR- 56/min, BP- 140/100mm Hg. The patient has been suffering fromamenorrhea for 5 months, has constipations.TSH- 28 mcIU/l (norm is 0,32-5). Craniogramshows no pathology. What is the etiology ofobesity?

A. HypothyroidB. Hypo-ovarianC. Hypothalamic-pituitaryD. Alimentary and constitutiveE. Hypercorticoid

119. A patient complains of painless ”sores”on his penis and inguinal lymph nodesenlargement. Synthomycin emulsion that thepatient have been applying to the ”sores” wasineffective. Objectively: on the inner leaf ofthe foreskin there are three closely situatedrounded erosions, 0,5 cm in diameter, withdense infiltration that can be palpated at theirbases. Make the preliminary diagnosis:

A. Primary syphilisB. Herpes simplex (Herpes pro genitalis)C. Candidiasis of the inner leaf of the foreskinD. ShinglesE. Erythema multiforme

120. A 1,5-month-old child on breasfeedi-ng presents from birth with daily vomiting,irregular liquid foamy feces, and meteori-sm, which are resistant to antibacterial andprobiotic therapy; no increase of body massis observed. The child’s condition improved,when breastmilk was substituted with ”NANlow lactose” formula. What pathology is it?

Page 18: Department of human resources policy, education …D. Pelvioperitonitis E. Incipient abortion 5. 5 weeks after hypothermia a 22-year-old patient developed fever, weakness, muscle pain,

Крок 2 Medicine (англомовний варiант, iноземнi студенти) 2017 рiк 16

A. Lactase deficiencyB. Intestinal lambliasis (Giardiasis)C. Infectious enteritisD. Drug-induced enteritisE. Functional dyspepsia

121. A 12-year-old girl after a case of respi-ratory infection developed dyspnea at rest,paleness of skin. Heart rate is 110/min., BPis 90/55 mm Hg. Heart sounds are muffled.Borders of relative heart dullness: right - theparasternal line, upper - the III rib, left -1,0 cm outwards from the midclavicular line.Make the provisional diagnosis:

A. Infectious myocarditisB. Functional cardiopathyC. Somatoform autonomic dysfunctionD. Hypertrophic cardiomyopathyE. Exudative pericarditis

122. A 48-year-old man complains of fati-gue, excessive sweating, severe skin itchi-ng, undulant fever, enlarged cervical andsupraclavicular lymph nodes. Objectively:paleness of skin and mucosa, cervical lymphnodes are mobile, dense, elastic, walnut-sized,painless, not attached to the skin. Completeblood count: erythrocytes - 3, 0 · 1012/l, Hb-100 g/l, leukocytes - 14 · 109/l, eosinophils -6%, basophils - 3%, band neutrophils - 11%,segmented neutrophils - 69%, lymphocytes -7, monocytes - 4%, platelets - 280 · 109/l, ESR-37 mm/hour. What method should be appliedto verify the diagnosis?

A. Lymph node biopsyB. Sternal punctureC. Muscle biopsyD. Chest X-rayE. Lumbar puncture

123. A patient after a blow to the headdeveloped general symptoms of cerebral di-sturbance, nausea, vomiting, focal signs -hemi-hyperreflexia S>D, hemihyperesthesiaon the left, marked meningeal syndrome. Nei-ther cranial X-ray nor computer tomographyrevealed any pathologies. What examinationmethod would allow making and clarificationof the diagnosis?

A. Lumbar punctureB. EchoencephalographyC. ElectroencephalographyD. AngiographyE. Pneumoencephalography

124. A 72-year-old man complains of lowerextremity edema, sensation of heaviness inthe right subcostal area, dyspnea of rest.For over 25 years he has been sufferi-ng from COPD. Objectively: orthopnea,jugular venous distention, diffuse cyanosis,acrocyanosis. Barrel chest is observed,on percussion there is vesiculotympanitic(bandbox) resonance, sharply weakened vesi-cular respiration on both sides, moist crepitant

crackles in the lower segments of the lungs.Heart sounds are weakened, the II heartsound is accentuated over the pulmonaryartery. The liver is +3 cm. What complicatedthe clinical course of COPD in this patient?

A. Chronic pulmonary heartB. Pulmonary embolismC. Acute left ventricular failureD. Diffuse pneumosclerosisE. Community-acquired pneumonia

125. A multigravida at 39 weeks of gestationpresenting with regular labour activity for 8hours has been delivered to a hospital; thewaters broke an hour ago. She complains ofheadache, seeing spots. BP is 180/100 mm Hg.Urine test results: protein - 3,3 g/l, hyalinecylinders. Fetal heart rate is 140/min, rhythmi-cal. Vaginal examination reveals completecervical dilatation, the fetal head is on thepelvic floor, sagittal suture is in line withobstetric conjugate, the occipital fontanel isunder the pubis. What is the optimal tactics oflabour management?

A. Outlet forcepsB. Cavity forcepsC. Cesarean sectionD. Vacuum extraction of the fetusE. Conservative labour management

126. After the contact with chemicals a plantworker has suddenly developed stridor, voi-ce hoarseness, barking cough, progressi-ng dyspnea. Objective examination revealsacrocyanosis. What is the provisional di-agnosis?

A. Laryngeal edemaB. Laryngeal carcinomaC. PATED. Pulmonary atelectasisE. Pneumothorax

127. An 18-year-old woman complains of painin her lower abdomen, profuse purulent di-scharge from the vagina, temperature rise upto 37,8oC . Anamnesis states that she had arandom sexual contact the day before the si-gns appeared. She was diagnosed with acutebilateral adnexitis. On additional examinati-on: leukocytes are present throughout all visi-on field, bacteria, diplococci with intracellularand extracellular position. What is the mostlikely agent in the given case?

A. Neisseria gonorrhoeaeB. Escherichia coliC. Chlamydia trachomatisD. Trichomona vaginalisE. Staphylococcus aureus

128. A 58-year-old man complains of generalweakness, loss of 10 kg of weight within 1,5months, progressive pain in the lumbar regi-on, increased blood pressure up to 220/160mm Hg, low grade fever. Objectively: in the

Page 19: Department of human resources policy, education …D. Pelvioperitonitis E. Incipient abortion 5. 5 weeks after hypothermia a 22-year-old patient developed fever, weakness, muscle pain,

Крок 2 Medicine (англомовний варiант, iноземнi студенти) 2017 рiк 17

right hypochondrium deep palpation reveals aformation with uneven surface and low mobi-lity; veins of the spermatic cord and scrotumare dilated. Blood test results: Hb- 86 g/l,ESR- 44 mm/h. Urine test results: specificgravity - 1020, protein - 0,99 g/l, RBC coverthe whole field of vision, WBC- 4-6 in the fieldof vision. What is the provisional diagnosis?

A. Renal tumourB. UrolithiasisC. Acute pyelonephritisD. Acute glomerulonephritisE. Nephroptosis

129. Anamnesis of a 30-year-old patientincludes closed thoracic injury. Lately thepatient has been suffering from increasingdyspnea, sensation of heaviness in the rightsubcostal area, and heart rate disturbances.Objectively: acrocyanosis, bulging cervicalveins, ascites, edema of the lower extremiti-es. Heart auscultation reveals muffled heartsounds, additional III heart sound is detected.Provisional diagnosis of constrictive peri-carditis was made. What diagnostic techniquewould NOT confirm the diagnosis?

A. US of abdomenB. Computer tomographyC. EchocardiographyD. Magnetic resonance imagingE. Chest X-ray

130. A 72-year-old woman suffers from di-abetes mellitus type 2, concomitant diseasesare stage 2 hypertension and stage 2B heartfailure. She takes metformin. Hypertensic cri-sis had occurred the day before, after whi-ch the patient developed extreme weakness,myalgias, thirst, dry mouth, polyuria. BPis 140/95 mm Hg, heart rate is 98/min.,no edemas or smell of acetone detected.What measures should be taken to preventdevelopment of comatose state in the patient?

A. Stop metformin, prescribe short-actinginsulinB. Double the dosage of metforminC. Apply hypotonic solution of sodium chlori-deD. Additionally prescribe long-acting insulinE. Prescribe glibenclamide

131. The body of a 24-year-old woman withprobable signs of poisoning has been found onthe street. Forensic medical examination wasrequested by an investigator during exami-nation of the site and the body. Accordingto the Criminal Procedure Code currently inforce in Ukraine, forensic medical examinati-on is required when it is necessary to determi-ne the:

A. Cause of deathB. Manner of deathC. Time of deathD. Mode of deathE. Mechanism of death

132. It is the 3rd day after the normal termlabor; the infant is rooming-in with themother and is on breastfeeding. Objectively:the mother’s general condition is satisfactory.Temperature is 36,4oC , heart rate is 80/min.,BP is 120/80 mm Hg. Mammary glandsare soft and painless; lactation is moderate,unrestricted milk flow. The uterus is dense,the uterine fundus is located by 3 fingers widthbelow the navel. Lochia are sanguino-serous,moderate in volume. Assess the dynamics ofuterine involution:

A. Physiological involutionB. SubinvolutionC. LochiometraD. Pathologic involutionE. Hematometra

133. Survey radiograph of a 52-year-oldworker of an agglomeration plant (28-year-long record of service, the concentrationof metal dust is 22-37 mg/m3) shows mi-ldly pronounced interstitial fibrosis with di-ffused contrasting well-defined small nodularshadows. The patient has no complaints.Pulmonary function is not compromised.What is the provisional diagnosis?

A. SiderosisB. SilicosisC. Anthraco-silicatosisD. SilicatosisE. Anthracosis

134. A 53-year-old woman complains of wei-ght loss up to 10 kg within the last 2 years,liquid foul-smelling stool two times a day thatpoorly washes off the toilet, periodic boutsof nausea, girdle pain in the upper abdomen.Objectively: pain in Gubergrits zone (on theright from navel) and at Mayo-Robson’s poi-nt. Biochemical blood analysis: glucose - 3,2mmol/l, bilirubin - 16,5 mcmol/l, crude protein- 56,4 g/l. Urine diastase/amylase - 426 g/h/l.D-xylose test (oral administration of 25 g ofd-xylose) after 5 hours reveals 3 g of xylose inurine. The most likely diagnosis is:

A. Pancreatitis. Malabsorption syndromeB. Pseudomembranous colitisC. Nonspecific ulcerative colitisD. Irritable bowel syndromeE. Chronic gastritis

135. A 15-year-old teenager has undergonemedical examination in military recruitmentcenter. The following was revealed: intervalsystolic murmur at the cardiac apex, accentof the II heart sound over the pulmonaryartery, tachycardia. What additional exami-nation method will be the most informative

Page 20: Department of human resources policy, education …D. Pelvioperitonitis E. Incipient abortion 5. 5 weeks after hypothermia a 22-year-old patient developed fever, weakness, muscle pain,

Крок 2 Medicine (англомовний варiант, iноземнi студенти) 2017 рiк 18

for determining diagnosis?

A. EchocardiographyB. ElectrocardiographyC. X-rayD. PhonocardiographyE. Rheography

136. A 64-year-old patient has been hospitali-zed with complaints of progressive jaundicethat developed over 3 weeks without painsyndrome and is accompanied by generalweakness and loss of appetite. Objectively:temperature is 36,8oC , heart rate is 78/min,abdomen is soft and painless, peritoneum irri-tation symptoms are not detected, palpati-on reveals sharply enlarged tense gallbladder.What disease can be characterised by thesesymptoms?

A. Cancer of pancreas headB. Duodenal ulcerC. Acute cholecystitisD. Chronic cholecystitisE. Cholecystitis caused by lambliasis

137. A 6-year-old girl came to a general practi-tioner with her mother. The child complainsof burning pain and itching in her externalgenitalia. The girl was taking antibiotics theday before due to her suffering from acutebronchitis. On examination: external genitaliaare swollen, hyperemic, there is white depositaccumulated in the folds. The most likely di-agnosis is:

A. Candidal vulvovaginitisB. TrichomoniasisC. Nonspecific vulvitisD. Helminthic invasionE. Herpes vulvitis

138. A 37-year-old patient complains of painin the spinal column, reduced mobility. Thecondition persists for 7 years. ”Sway back” isobserved, there is no movement in all spi-nal regions. On X-ray: ”bamboo spine” isdetected. What is the most likely diagnosis?

A. Ankylosing spondylitisB. OsteochondrosisC. Spondylitis deformansD. Tuberculous spondylitisE. Spondylolisthesis

139. A 25-year-old patient is not married andhas sexual relations with several partners.During the last 3 months he noticed a smallamount of mucoserous discharge from theurethra. Subjectively: periodical itching orburning pain in the urethra. Two months agopain in the knee joint developed. Possibilityof trauma or exposure to cold is denied by thepatient. During the last week eye discomfortis noted - lacrimation and itching. What provi-sional diagnosis can be made?

A. Reactive arthritisB. Rheumatoid arthritisC. Seasonal pollinosisD. Bacterial nonspecific urethral conjunctivitisE. Upper respiratory tract infection (URTI)that affects conjunctiva and joints

140. A 19-year-old woman complains ofsevere pain in the axillary crease. Conditi-on onset occurred a week ago after her swi-mming in a cold river and epilation. The nextday a painful ”boil” appeared. The ”boil” wasincreasing in size every day and became aplum-sized tumor. Upon examination thereare nodular conical growths joined togetherdetected, the skin covering them is bluish-redin color. Some nodules have fistulous openi-ngs producing thick purulent mass. Bodytemperature is 38, 5oC , general malaise. Whatis the most likely diagnosis?

A. HydradenitisB. CarbuncleC. Cutaneous tuberculosisD. Necrotizing ulcerative trichophytosisE. Pyoderma chancriformis

141. A woman complains of weight gain,chills, edema, xeroderma, somnolence, diffi-culties with focusing. Objectively: height is165 cm; weight is 90 kg; body proportionsare of female type, to- 35,8oC , heart rate -58/min, BP- 105/60 mm Hg. Heart soundsare weakened, bradycardia is observed. Otherinternal organs have no alterations. Thyroidgland cannot be palpated. Milk secretionfrom mammary glands is observed. Hormonetest revealed increased levels of thyroid-stimulating hormone (TSH) and prolactin,and decreased level of thyroxine (�4). Whatis the cause of obesity?

A. Primary hypothyroidismB. Secondary hypothyroidismC. ProlactinomaD. HypopituitarismE. Adiposogenital dystrophy

142. ECG revealed the following in a 10-year-old child: sharp acceleration of the heartrate - 240/min., P wave overlaps with T waveand deforms it, moderate lengthening of PQinterval, QRS complex is without alterations.What pathology does this child have?

A. Paroxysmal atrial tachycardiaB. Atrial hypertrophyC. Ventricular hypertrophyD. WPW syndromeE. Extrasystole

143. A 54-year-old patient complains ofweakness, jaundice, itching skin. Diseaseonset was 1,5 months ago: fever up to 39oCappeared at first, with progressive jaundicedeveloped 2 weeks later. On hospitalizati-on jaundice was severely progressed. Livercannot be palpated. Gallbladder is enlarged

Page 21: Department of human resources policy, education …D. Pelvioperitonitis E. Incipient abortion 5. 5 weeks after hypothermia a 22-year-old patient developed fever, weakness, muscle pain,

Крок 2 Medicine (англомовний варiант, iноземнi студенти) 2017 рiк 19

and painless. Blood bilirubin is 190 mcmol/l(accounting for direct bilirubin). Stool isacholic. What is the most likely jaundicegenesis in this patient?

A. Mechanical jaundiceB. Hepatocellular jaundiceC. Hemolytic jaundiceD. Caroli syndromeE. Gilbert’s syndrome

144. During hemotransfusion the patientdeveloped nausea, tremor, lumbar andretrosternal pain. On examination the skinis hyperemic, later developed pallor; the pati-ent presents with hyperhidrosis, labored respi-ration, pulse is 110/min., BP is 70/40 mm Hg.Urine is black colored. What complicationdeveloped in the patient?

A. Posttransfusion shockB. Acute renal failureC. Pulmonary embolismD. Anaphylactic shockE. Hypotonic crisis

145. A 22-year-old man suddenly developedextreme weakness, nausea, vomiting withtraces of blood. The patient is known tosuffer from peptiv ulcer disease of duodenumand hemophilia A. Objectively: heart rate -102/min., BP- 100/60 mm Hg. Complete bloodcount: erythrocytes - 3, 2 · 1012/l, Hb- 98 g/l,color index - 0,92, leukocytes - 7, 4 · 109/l,platelets - 240 · 109/l, ESR- 11 mm/hour. Whatmeasure would most effectively decreasehemorrhaging in this case?

A. CryoprecipitateB. Aminocapronic acidC. Native plasmaD. Direct transfusion of donor bloodE. Platelet concentrate transfusion

146. A 22-year-old woman complains ofamenorrhea for 8 months. Anamnesis statesthat menarche occured at the age of 12,5. Si-nce the age of 18 the patient has a history ofirregular menstruation. The patient is nulli-gravida. The mammary glands are developedproperly, nipples discharge drops of milkwhen pressed. Hormone test: prolactin levelis 2 times higher than normal. CT reveals abulky formation with diameter of 4 mm inthe region of sella. What is the most likely di-agnosis?

A. Pituitary tumourB. Lactational amenorrheaC. Stein-Leventhal syndrome (polycystic ovarysyndrome)D. Sheehan’s syndrome (postpartum hypopi-tuitarism)E. Cushing’s disease

147. A 13-year-old girl complains of fatigabili-ty, frequent headaches, cardialgia. Eight yearsago she had a case of pyelonephritis. Urine

analyses periodically revealed leukocyturia.The child has undergone no further treatment.On examination: increased BP up to 150/100mm Hg. Ultrasound investigation revealed si-gnificant reduction of the right kidney. Whatprocess is leading in arterial hypertensionpathogenesis in this case?

A. Hyperactivity of renin-angiotensin systemB. Disturbance of water-electrolytic balanceC. Disturbance of renal circulationD. HypersympathicotoniaE. Increased cortisol level

148. A 3-month-old child presents withsaffron-yellow coloring of the skin, sclera,and mucous membranes. The abdomen isenlarged, hepatomegaly and splenomegalyare observed. In blood there is conjugatedbilirubin-induced hyperbilirubinemia. Onintravenous cholangiocholecystography:opacified bile is discharged into the intesti-ne. Transaminase activity is normal. What isthe most likely diagnosis?

A. Biliary atresiaB. Physiologic jaundiceC. Hemolytic disease of newbornD. Crigler-Najjar syndromeE. Congenital hepatitis

149. A 51-year-old woman complains ofheadache, trembling, paresthesiae, palpi-tations, increased blood pressure up to280/160 mm Hg. The day before she experi-enced exhausting headache, vascular pulsati-on, palpitations, asphyxia, stomachache,unbearable fear of coming death. The pati-ent paled and broke out in cold sweat. Inurine there is increased content of vani-llylmandelic acid. What disease causes suchclinical presentation in the patient?

A. PheochromocytomaB. Conn’s syndrome (primaryhyperaldosteronism)C. Cushing’s syndromeD. Primary hypertensionE. Cushing’s disease

150. A 52-year-old patient suffers frommarked dyspnea during physical exertion,non-productive cough. The patient’s condi-tion has been persisting for 8 months. Thepatient has been a smoker for 30 years. Inthe lungs there are cellophane-type cracklesauscultated on both sides. Respiration rate is26/min., oxygen saturation of blood is 92%.On spirometry: moderate restrictive-type di-sturbance of external respiration. What is themost likely diagnosis?

Page 22: Department of human resources policy, education …D. Pelvioperitonitis E. Incipient abortion 5. 5 weeks after hypothermia a 22-year-old patient developed fever, weakness, muscle pain,

Крок 2 Medicine (англомовний варiант, iноземнi студенти) 2017 рiк 20

A. Idiopathic fibrosing alveolitisB. Chronic obstructive pulmonary disease(COPD)C. Chronic bronchitisD. Community-acquired pneumoniaE. Sarcoidosis

151. A 26-year-old patient with affective bi-polar disorder has developed a conditionmanifested by mood improvement, behavi-oural and sexual hyperactivity, verbosity, acti-ve body language, reduced need for sleep.Which of the following drugs would be mosteffective in this case?

A. Neuroleptics with sedative effectB. Antidepressants with activating effectC. Neuroleptics with activating effectD. TranquilizersE. Antidepressants with sedative effect

152. A 19-year-old patient complains ofdyspnea on exertion. He often has bronchitisand pneumonia. Since childhood the patientpresents with cardiac murmur. Auscultationrevealed splitting of the II sound above thepulmonary artery, systolic murmur in the 3rdintercostal space at the left sternal border.ECG detected right bundle branch block.What is the provisional diagnosis?

A. Atrial septal defectB. Open ductus arteriosusC. AortarctiaD. Aortic stenosisE. Mitral insufficiency

153. A 30-year-old patient was in a car acci-dent. He is unconscious, pale, has threadypulse. In the middle third of the right thighthere is an extensive laceration with ongoi-ng profuse external arterial bleeding. Whaturgent actions must be taken to save the lifeof the patient?

A. Tourniquet above the wound of the rightthighB. Tourniquet below the wound of the rightthighC. Artificial lung ventilationD. Precordial thumpE. Application of plaster bar

154. A 25-year-old patient has been admi-tted to the hospital with the followingproblems: weakness, sweating, itching, wei-ght loss, enlarged submandibular, cervical,axillary, inguinal lymph nodes. Objectively:hepatomegaly. Lymph node biopsy revealedgiant Berezovsky-Reed-Sternberg cells,polymorphocellular granuloma composed oflymphocytes, reticular cells, neutrophils, eosi-nophils, fibrous tissue, and plasma cells. Whatis the most likely diagnosis?

A. Lymphogranulomatosis (Hodgkin’slymphoma)B. Lymph node tuberculosisC. LymphoreticulosarcomaD. Cancer metastases to the lymph nodesE. Macofollicular reticulosis

155. An 9-year-old child was hospitalizedfor fever up to 39,8oC , inertness, moderateheadache, vomiting. Examination revealedmeningeal symptoms. Lumbar puncture wasperformed. The obtained fluid was characteri-sed by increased opening pressure, wastransparent, with the cell count of 450 cells per1 mcL (mainly lymphocytes - 90%), glucoselevel of 3,6 mmol/l. What agent could havecaused the disease in the child?

A. EnterovirusB. Neisseria meningitidisC. Mycobacterium tuberculosisD. Staphylococcus aureusE. Streptococcus pneumoniae

156. A 25-year-old woman has a self-detectedtumor in the upper outer quadrant of her ri-ght breast. On palpation there is a painlessfirm mobile lump up to 2 cm in diameter, peri-pheral lymph nodes are without alterations. Inthe upper outer quadrant of the right breastultrasound revealed a massive neoplasm withincreased echogenicity sized 21x18 mm. Whatis the most likely diagnosis?

A. FibroadenomaB. LactoceleC. Diffuse mastopathyD. Mammary cancerE. Mastitis

157. A 68-year-old man complains of inabili-ty to urinate for a day. On attempt of urinarybladder catheterization there was detected arough stricture in the membranous portion ofthe urethra. What first aid tactics should beapplied in this case?

A. Troacar cystostomyB. AdenomectomyC. Optical internal urethrotomyD. α-adrenoblockersE. Urinary bladder catheterization

158. Heart X-ray of a 31-year-old man hasrevealed the following: with tightly filledopacified esophagus there is a marginal filli-ng defect in its middle third on the posteriorwall; the defect is 1,8x1,3 cm in size with clearoval border. Mucosal folds are retained andenvelop the defect; wall peristalsis and elasti-city are not affected. There are no complai-nts regarding the condition of the patient’salimentary canal. Make the provisional di-agnosis:

Page 23: Department of human resources policy, education …D. Pelvioperitonitis E. Incipient abortion 5. 5 weeks after hypothermia a 22-year-old patient developed fever, weakness, muscle pain,

Крок 2 Medicine (англомовний варiант, iноземнi студенти) 2017 рiк 21

A. Esophageal tumorB. Achalasia cardiaeC. Esophageal burnsD. DiverticulumE. Barrett esophagus

159. A patient with signs of generaloverexposure to cold presenting with localfrostbites of fingers has been delivered intoan admission room. Objectively: conscious,inert, speech is slow, the skin of the face iscold, body temperature is 34oC , heart rate is68/min. What would be the actions of a doctoron call?

A. Hospitalize the patient to the surgicaldepartmentB. Hospitalize the patient to the therapeuticsdepartmentC. Hospitalize the patient to the traumatologydepartmentD. Let the patient go homeE. Refer to a family doctor on the next day

160. A 22-day-old infant developedsubcutaneous red nodes from 1,0 to 1,5 cm insize on the scalp; later the nodes suppurated.Temperature increased up to 37,7oC , intoxi-cation symptoms appeared, regional lymphnodes enlarged. Complete blood count:anemia, leukocytosis, neutrocytosis, increasedESR. What diagnosis will you make?

A. PseudofurunculosisB. PemphigusC. VesiculopustulosisD. Scalp phlegmonE. -

161. During examination of a healthy infant,the child takes a toy into his hands, turnsfrom the back to the side; when lying on thestomach he can firmly prop himself up on hisforearms; the child laughs and makes joyfulexclamations. The age of the child is:

A. 4 monthsB. 2 monthsC. 5 monthsD. 1 monthE. 6 months

162. After significant physical exertion a 66-year-old man with deep vein thrombosis ofthe extremities developed shortness of breath,intense pain in the chest on the left, markedpalpitations. The patient’s condition is grave,his face is cyanotic, the cervical veins areswollen, BP is 60/40 mm Hg. What investi-gation method would be the most advisable inthis case?

A. Selective angiopneumographyB. Chest X-rayC. EchocardiographyD. Magnetic resonance imaging of the chestE. Fiber-optic bronchoscopy

163. A 32-year-old woman complains of

general fatigue, low-grade fever persistingfor 4 months, lumbar pain, and dysuria.Anamnesis includes frequent acute respi-ratory diseases, overexposure to cold, low-calorie diet, a case of pulmonary tuberculosisin childhood. Clinical urine analysis: pH-4,8, leukocyturia, hematuria. Complete bloodcount: leukocytosis, lymphocytosis, increasedESR. Urography concludes: dilatation ofrenal pelvis and calyceal system of both ki-dneys, foci of calcification in the projection ofright kidney parenchyma. What is the mostlikely diagnosis?

A. NephrotuberculosisB. Right renal cystC. Right renal carcinomaD. Acute glomerulonephritisE. Chronic pyelonephritis

164. A full-term newborn (born with the bodyweight of 3900 g at gestational age of 39weeks) on the first day of his life developedrespiratory disturbances: dyspnea, arrhythmicrespiration, cyanosis attacks. On examinationthere is paradoxical respiration observed andleft side of the chest lags behind in the act ofbreathing. On auscultation the respiration isweakened in the lungs on the left. Neurologi-st diagnosed the patient with left-sided Erb-Duchenne palsy. Complete blood count showsno changes. What is the most likely diagnosis?

A. Left-sided diaphragm paresisB. Congenital pneumoniaC. Left-sided pneumothoraxD. Respiratory distress syndromeE. Transient tachypnea of the newborn

165. A 12-year-old child had three attacks ofacute rheumatic fever accompanied by cardi-tis. Examination revealed the symptoms ofchronic tonsillitis, mitral insufficiency, andcarious teeth. What is the optimal methodof secondary prophylaxis?

A. Year-round bicillin prophylaxis until the ageof 25B. Course of cardiotrophic drugs twice a yearC. Year-round bicillin prophylaxis for 3 yearsD. TonsillectomyE. Oral cavity sanitation

166. A 24-year-old patient had been deliveredto a thoracic department with a chest injury,fracture of the IV, V, VI ribs on the right.Plan radiography showed the fluid level inthe pleural cavity to be reaching the III ribon the right. Puncture contained blood clots.What is the optimal treatment tactics?

A. Emergency thoracotomyB. Pleural punctureC. Thoracentesis and thoracostomyD. Hemostatic therapyE. Medical thoracoscopy

167. A 3-year-old child has been delivered to

Page 24: Department of human resources policy, education …D. Pelvioperitonitis E. Incipient abortion 5. 5 weeks after hypothermia a 22-year-old patient developed fever, weakness, muscle pain,

Крок 2 Medicine (англомовний варiант, iноземнi студенти) 2017 рiк 22

a hospital with complaints of pain in the legs,fever, loss of appetite. Objectively: pale skinand mucosa, hemorrhagic rash. Lymph nodesare enlarged, painless, dense and elastic, notmatted together. Bones, joints, and abdomenare painful. The liver and spleen are enlarged.Hemogram: Hb- 88 g/l, color index - 1,3,platelets - 80 · 109/l, leukocytes - 25, 8 · 109/l,lymphoblasts - 70%, ESR- 52 mm/hour. Makethe provisional diagnosis:

A. Acute leukemiaB. Thrombocytopenic purpuraC. Acute rheumatic feverD. Infectious mononucleosisE. Hemorrhagic vasculitis (Henoch-Schonleinpurpura)

168. A 60-year-old man has a diet consisting ofunvaried food staples: mostly cereals, potato,pasta; few vegetables and little fats (especiallyanimal fats). During medical examination hecomplains of deterioration of his twilight visi-on. This condition can be caused by lack of:

A. RetinolB. Amino acidsC. FatsD. CalciumE. Carbohydrates

169. A 14-year-old girl came to a generalpractitioner with complaints of weakness,loss of appetite, headache, rapid fatigabili-ty. Her last menstruation was profuse andlasted for 14 days after previous delay of 2months. Objectively: the skin is pale, heartrate is 90/min., BP is 110/70 mm Hg, Hb is88 g/l. Rectal examination: the uterus and itsappendages are without changes, no dischargefrom the genital tracts. What complicationoccurred in the patient?

A. Posthemorrhagic anemiaB. Somatoform autonomic dysfunction ofhypotonic typeC. MigraineD. GastritisE. Dysmenorrhea

170. A postpartum woman on the 12th dayafter the normal delivery complains of painlocalized in her left gastrocnemius muscle.Body temperature is 37,2oC ; pulse is 85/min,rhythmic; blood pressure is 128/80 mm Hg.Mammary glands are soft and painless. Theuterus is behind the pubis. The left leg in thearea of gastrocnemius muscle is by 3 cm largerthan the right leg in the diameter. Internalorgans present no pathologies. What compli-cation can be suspected?

A. Deep vein thrombosis of the shinB. Iliofemoral thrombosisC. Varicose veins of lower extremitiesD. EndometritisE. Myositis

171. Examination of a Rh-negative pregnantwoman at 32 weeks of gestation revealeda four-time rise of Rh-antibody titer within2 last weeks; the titer is 1:64. The first twopregnancies resulted in antenatal fetal deathdue to hemolytic disease. What is the optimaltactics of pregnancy management?

A. Early deliveryB. Delivery at 37 weeks of gestationC. Screening for Rh-antibodies in 2 weeks andurgent delivery in case of further increase ofantibody titerD. Introduction of anti-Rh (D)immunoglobulinE. Ultrasound for signs of hemolytic disease ofthe fetus

172. During last several weeks an 11-year-old girl has been complaining of dyspnea andedema of shins and feet after physical exerci-se. After a long rest or sleep through the nighther edemas diminish significantly. On clini-cal examination there are enlarged liver andrasping systolic murmur over the cardiac area.Blood and urine analyses are without changes.What is the most likely cause of the child’sedema?

A. Heart failureB. Angioneurotic edemaC. Acute pyelonephritisD. HepatocirrhosisE. Nephrotic syndrome

173. A 74-year-old man visited an urologistwith complaints of pain above the pubis andinability to urinate for 8 hours. At home hehad taken antispasmodics and had a warmbath but no improvement occurred. Objecti-vely: the abdomen is soft and painful abovethe pubis; dullness of percussion sound isobserved above the pubis. Murphy’s punch si-gn (costovertebral angle tenderness) is negati-ve on both sides. What condition does thepatient have?

A. Acute urinary retentionB. Paradoxal ischuriaC. Chronic urinary retentionD. AnuriaE. Oliguria

174. During preventive examination a 58-year-old man on chest X-ray presents with multi-ple globular pale shadows 3 cm in diameterwithin parenchyma of the both lungs. Exami-nation in the oncologic hospital: the primaryfocus is not found; transbronchial biopsy wi-th cytologic investigation detected cells ofglandular neoplasm. What tactics should thephysician choose?

Page 25: Department of human resources policy, education …D. Pelvioperitonitis E. Incipient abortion 5. 5 weeks after hypothermia a 22-year-old patient developed fever, weakness, muscle pain,

Крок 2 Medicine (англомовний варiант, iноземнi студенти) 2017 рiк 23

A. Polychemotherapy coursesB. Exploratory laparotomyC. Exploratory thoracotomyD. LaparoscopyE. Symptomatic treatment at home

175. A 36-year-old man developed a diseasewith acute onset 6 hours ago. The patientpresents with pain in the epigastric, ileocecal,and paraumbilical areas, vomiting, weakness,nausea, and body temperature of 38,5oC .Stool is liquid, profuse, frequent, retains fecalnature, foul-smelling, frothy, colored darkgreen. The stomach is moderately distendedand painful on palpation. The patient attri-butes his disease to eating raw chicken eggsone day before the clinical signs of the diseaseappeared. What is the most likely diagnosis?

A. SalmonellosisB. ShigellosisC. Typhoid feverD. CholeraE. Food toxicoinfection

176. A 55-year-old woman came to agynecologist with complaints of leukorrheaand bloody discharge from the vagina after5 years of menopause. Anamnesis statesno pregnancies. Bimanual examination: theuterus and uterine appendages are withoutchanges. During diagnostic curettage of theuterine cavity the physician scraped offenchephaloid matter. What is the most likelydiagnosis in this case?

A. Endometrial carcinomaB. AdenomyosisC. Subserous uterine myomaD. Cervical carcinomaE. Ovarian carcinoma

177. A 10-year-old boy is delivered into apolytrauma unit after he received a blunttrauma of the thorax, having fallen fromthe bicycle. Upon hospitalization his bloodpressure is 110/80 mm Hg, heart rate is 96/min.Chest X-ray is noncontributive to the di-agnosis. Echocardiogram shows free liquidin the pericardial cavity, in the amount of upto 100 ml. In an hour after the hospitalizationthe patient started to develop increasing si-gns of heart failure: jugular venous distention,decreased blood pressure down to 90/70 mmHg, tachycardia up to 120/min. On auscultati-on muffled heart sounds. What would be theprimary tactics of a physician?

A. PericardiocentesisB. Cardiac glycosides intravenouslyC. Constant oxygenotherapyD. Diuretics intravenouslyE. Antibiotics intravenously

178. After the celebratory feast that tookplace the day before, a 35-year-old man washospitalized with complaints of marked painwithin the I metatarsophalangeal articulati-

on on the right, which developed late inthe night, and impaired walking. Objecti-vely: the metatarsophalangeal articulationis swollen, hyperemic, hot to touch, pai-nful on movement. In blood: erythrocytes -5, 1·1012/l, Нb- 155 g/l, leukocytes- 13, 0·109/l,ESR- 50 mm/hour, CRP- 46 mg/dl, uricacid - 720 mcmol/l. X-ray of feet articulati-ons: osteoporosis, narrowing of interarti-cular spaces, numerous punched-out erosions.Make the preliminary diagnosis:

A. GoutB. OsteoarthritisC. Reactive arthritisD. Rheumatoid arthritisE. Psoriatic arthritis

179. A woman in her early- to mid-thirtieshas lost her consciousness 3-5 minutes ago.On examination: the skin is pale, no pulseover the carotid arteries, no spontaneousrespiration, pupils are dilated; the patient isnonresponsive, presents with atony. The pati-ent’s condition can be determined as:

A. Apparent deathB. Natural deathC. SyncopeD. Brain deathE. Coma

180. A 32-year-old woman complains of bodyweight loss despite her increased appetite,nervousness, and tremor of the extremities.Objectively: the skin is moist; the thyroidgland is diffusely enlarged, painless, soft, andmobile. Blood test: increased level of T3,T4, and thyroid-stimulating hormone (THS).What is the most likely diagnosis?

A. Diffuse toxic goiterB. Thyroid carcinomaC. Autoimmune (Hashimoto’s) thyroiditisD. Thyroid adenomaE. Diffuse nontoxic goiter

181. A 57-year-old patient complains ofsensation of dryness and pain duringswallowing, frequent unbearable cough, thevoice is hoarse. Disease onset was abrupt. Onlaryngoscopy: laryngeal mucosa is hyperemic,vocal folds are swollen, laryngeal lumencontains viscous secretion. What diagnosis isit?

A. Acute laryngitisB. Acute stenosing laryngotracheitisC. Bronchial asthmaD. Flegmonous laryngitisE. Laryngeal diphtheria

182. A 24-year-old pregnant woman on her37th week of pregnancy has been delivered toa maternity obstetric service with complaintsof weak fetal movements. Fetal heartbeats are95/min. On vaginal examination the uterinecervix is tilted backwards, 2 cm long, external

Page 26: Department of human resources policy, education …D. Pelvioperitonitis E. Incipient abortion 5. 5 weeks after hypothermia a 22-year-old patient developed fever, weakness, muscle pain,

Крок 2 Medicine (англомовний варiант, iноземнi студенти) 2017 рiк 24

orifice allows inserting a fingertip. Biophysi-cal profile of the fetus equals 4 points. Whattactics of pregnancy management should bechosen?

A. Urgent delivery via a cesarean sectionB. Treatment of placental dysfunction andrepeated analysis of the fetal biophysicalprofile on the next dayC. Doppler measurement of blood velocity inthe umbilical arteryD. Urgent preparation of the uterine cervix fordeliveryE. Treatment of fetal distress, if ineffective,then elective cesarean section on the next day

183. During regular preventive gynecologi-cal examination a 30-year-old woman wasdetected to have dark blue punctulated”perforations” on the vaginal portion ofthe uterine cervix. The doctor suspectsendometriosis of the vaginal portion of theuterine cervix. What investigation methodwould be most informative for diagnosisconfirmation?

A. Colposcopy, target biopsy of the cervixB. US of small pelvisC. HysteroscopyD. Curettage of the uterine cavityE. Hormone testing

184. A 26-year-old woman came to agynecologist for a regular check-up. She hasno complaints. Per vaginum: the uterus lies inanteflexion, not enlarged, dense, mobile, pai-nless. On the left from the uterus in the area ofuterine appendages there is a mobile painlessoutgrowth that can be moved independentlyfrom the uterus. On the right the appendagescannot be detected. What additional investi-gation would be informative for diagnosisclarification?

A. US of lesser pelvisB. MetrosalpingographyC. Examination for urogenital infectionD. ColposcopyE. Colonoscopy

185. A 57-year-old patient complains ofdyspnea at rest. The patient presents withorthopnea, acrocyanosis, bulging cervical vei-ns. On percussion: dull sound over the lowerlung segments; on auscultation: no respiratorymurmurs. Heart rate is 92/min. Right-sidedcardiac dilatation is observed. The liver isenlarged by 7 cm. Shins are swollen. Pleuraleffusion is suspected. What indicator wouldconfirm the presence of transudate in thiscase?

A. Total protein content in the pleural fluidbelow 25 g/lB. Presence of atypical cellsC. Total protein content in the pleural fluidabove 30 g/lD. Specific gravity exceeding 1015E. Positive Rivalta’s test

186. A 28-year-old woman complains of gi-rdle pain in her epigastric and left subcostalareas with irradiation to the back, nausea,and vomiting without relief. On examinati-on a surgeon observes stomach distensionand meteorism. There are positive Mondor’s,Mayo-Robson’s, and Cullen’s symptoms.What is the most likely diagnosis?

A. Acute pancreatitisB. Acute cholecystitisC. Acute intestinal obstructionD. Aortic dissecting aneurysmE. Splenic infarction

187. A 36-year-old woman complains ofpain in her lumbar area, which irradiates toher lower right limb and increases duringmovements, and sensation of numbness inher limb. Objectively: palpation of the shinand thigh muscles is painful, positive stretchsymptom on the right. MRI scan: herniati-on of intervertebral disk L5-S1 4 mm in size.What is the most likely diagnosis?

A. Vertebrogenic lumbagoB. Vertebrogenic radicular syndrome of L5-S1on the rightC. Endarteritis of lower extremitiesD. Spinal strokeE. Acute myelitis

188. During routine medical examinati-on a 35-year-old woman presents withenlarged cervical and mediastinal lymphnodes. Her overall health is satisfactory.ESR is 30 mm/hour. Cervical node biopsywas performed. In the specimen there aregranulomas composed of epithelial and gi-ant cells, no caseous necrosis detected. Whatis the most likely diagnosis?

A. SarcoidosisB. LymphogranulomatosisC. Infectious mononucleosisD. Nonspecific lymphadenitisE. Lymph node tuberculosis

189. A 39-year-old woman complains of vi-olent pain in her left lumbar area with irradi-ation to the right iliac area. Several yearsago she was diagnosed with cholelithiasisand urolithiasis. The patient’s condition ismoderately severe, the skin is dry. Ortner’ssymptom is negative; costovertebral angletenderness is observed on the right. The mostreasonable treatment tactics would be:

Page 27: Department of human resources policy, education …D. Pelvioperitonitis E. Incipient abortion 5. 5 weeks after hypothermia a 22-year-old patient developed fever, weakness, muscle pain,

Крок 2 Medicine (англомовний варiант, iноземнi студенти) 2017 рiк 25

A. Spasmolytics and analgesicsB. Urgent hemodialysisC. Peritoneal dialysisD. Laparoscopic cholecystectomyE. Antibiotics

190. An 18-year-old patient complains of skinrash. The patient has been suffering from thiscondition for 5 years. The first instance of thisdisease occurred after a car accident. Objecti-vely: the patient presents with papular rashcovered in silvery scales, ”thimble” symptom(small pits on the nails), affected joints. Whatis the most likely diagnosis?

A. PsoriasisB. PanaritiumC. OnychomycosisD. Lupus erythematosusE. Rheumatism

191. A woman complains of frequent, li-quid stool (up to 9-10 times per day) withmucus and blood admixtures, dull pain in thehypogastrium, weight loss of 4 kg within thelast year. Objectively: malnutrition, dry skin,low turgor, aphthous stomatitis. The stomachis soft, the sigmoid colon is spastic and painfulon palpation. Occult blood test is positive. Fi-brocolonoscopy: edema, hyperemia, mucosalgranulation, pseudopolyps, small ulcers withirregular edges. Make the diagnosis:

A. Nonspecific ulcerative colitisB. Chronic enterocolitisC. Colon cancerD. Irritable bowel syndromeE. Crohn’s disease (regional enteritis)

192. A 48-year-old woman has been hospitali-zed due to development of tachysystolic atrialfibrillation. She has lost 5 kg of body weightwithin 2 months. On palpation there is a nodein the left lobe of the thyroid gland. Whatpathology resulted in the development of thiscondition?

A. Toxic nodular goiterB. Aterosclerotic cardiosclerosisC. Chronic thyroiditisD. Nontoxic nodular goiterE. Autoimmune thyroiditis

193. On the 9th day after childbirth theobstetric patient developed high fever upto 38oC . She complains of pain in the rightmammary gland. On examination a sharplypainful infiltrate can be palpated in the rightmammary gland, the skin over the infiltrateis red, subareolar area and nipple are swollenand painful. What is your diagnosis?

A. Abscess of the right mammary glandB. MastopathyC. Cancer of the right mammary glandD. Serous mastitisE. Fibrous cystic degeneration of the rightmammary gland

194. A 32-year-old woman complains of epi-sodes of intense fear that occur without visiblecause and last for 10-20 minutes; the episodesare characterized by rapid pulse, sweating,labored breathing, and vertigo. Specify the li-kely diagnosis:

A. Panic disorderB. Paranoid syndromeC. Manic syndromeD. Simple schizophreniaE. Claustrophobia

195. A resuscitation unit received a 46-year-old woman, who has been suffering fromdiabetes mellitus type 1 for approximately30 years. Objectively: the skin is pale, heartsounds are weakened, BP is 170/100 mmHg, lower limbs are markedly swollen. Bloodcreatinine - 1125 mcmol/l, urea - 49,6 mmol/l,potassium - 6.3 mmol/l, glucose - 7,6 mmol/l,glomerular filtration rate - 5 ml/min. Whattreatment is indicated for the patient in thefirst place?

A. HemodialysisB. Kidney transplantationC. HemofiltrationD. EnterosorptionE. Conservative detoxification therapy

196. A woman is on the 32nd week of hersecond pregnancy. She complains of fever,chills, nausea, vomiting, lumbar pain, anddysuria. Costovertebral angle tenderness ispresent on both sides. Urine analysis: pyuria,bacteriuria. Blood test: leukocytosis. What isthe most likely diagnosis?

A. Gestational pyelonephritisB. CystitisC. PyelitisD. GlomerulonephritisE. Latent bacteriuria

197. A 19-year-old young man complains ofcough with expectoration of purulent sputumin amount of 100 ml per day, hemoptysis,dyspnea, increased body temperature upto 37,8oC , general weakness, weight loss.The patient’s condition lasts for 4 years.Exacerbations occur 2-3 times a year.The patient presents with malnutrition,pale skin, cyanosis of the lips, drumstick(clubbed) fingers. Tympanic percussion soundin the lungs, weakened respiration, vari-ous numerous moist crackles in the lowerpulmonary segments on the left can beobserved. In blood: erythrocytes - 3, 2 · 1012/l,leukocytes - 8, 4 · 109/l, ESR- 56 mm/hour.On X-ray: lung fields are emphysematous, theleft pulmonary root is deformed and dilated.What is the most likely diagnosis?

Page 28: Department of human resources policy, education …D. Pelvioperitonitis E. Incipient abortion 5. 5 weeks after hypothermia a 22-year-old patient developed fever, weakness, muscle pain,

Крок 2 Medicine (англомовний варiант, iноземнi студенти) 2017 рiк 26

A. Multiple bronchiectasis of the left lungB. Chronic left-sided pneumoniaC. Chronic abscess of the left lungD. Left-sided pulmonary cystic dysplasiaE. Suppuration of the cyst in the left lung

198. A 37-year-old man working as atypesetter in a print shop complains ofrapid fatigability, paroxysmal attacks ofstomachache, weak drooping hands. Exami-nation of neurological status revealedhypotrophy of the forearm muscles.Carporadial reflexes are sharply weakened.Sensitivity is not disturbed. Gums presentwith dark blue border. What neurologicalpathology is it?

A. Lead polyneuropathyB. Guillain-Barre syndrome (postinfectiouspolyneuritis)C. ShinglesD. Ulnar neuropathyE. Brachial plexitis

199. A 54-year-old woman takes anti-hypertensive drugs for hypertension. Havi-ng discovered that her son was arrested andis under investigation, became agitated andextremely anxious. She lost her orientationin place, stopped recognizing her relatives,started hearing ”voices” threatening her and

her son with violence. She had opened herwindow (on the 8th floor) and tried to jumpout, resisted the people, who were holding herback. What drugs should be administered toterminate such condition of the patient?

A. AntipsychoticsB. AntidepressantsC. TranquilizersD. Nootropic agentsE. Antihypertensive drugs

200. A 43-year-old man, who has been abusi-ng alcohol and suffering from pulmonarytuberculosis, in the course of two weeksgradually developed general weakness,headache, diplopia, vomiting. Objectively:ptosis on the left, anisocoria S>D, exotropiaof the left eye, neck stiffness; Kernig’sand Brudzinski’s signs are positive. Incerebrospinal fluid: lymphocytic pleocytosis,low glucose, precipitation of cerebrospinalfluid resulted in production of fibrin film.What is the most likely diagnosis?

A. Tuberculous meningitisB. Subarachnoid hemorrhageC. Brainstem encephalitisD. Acute myelitisE. Basal arachnoiditis

Page 29: Department of human resources policy, education …D. Pelvioperitonitis E. Incipient abortion 5. 5 weeks after hypothermia a 22-year-old patient developed fever, weakness, muscle pain,

INSTRUCTIONAL BOOK

Testing Board

TEST ITEMS FOR LICENSING EXAMINATION: KROK 2. MEDICINE.

Kyiv. Testing Board.(English language).

Approved to print 24.04/№217. Paper size 60х84 1/8Offset paper. Typeface. Times New Roman Cyr. Offset print.Conditional print pages 24. Accounting publishing pages 28.Issue. 2610 copies

Page 30: Department of human resources policy, education …D. Pelvioperitonitis E. Incipient abortion 5. 5 weeks after hypothermia a 22-year-old patient developed fever, weakness, muscle pain,

List of abbreviationsList of abbreviations

ACTH Adrenocorticotrophic hormoneADP Adenosine diphosphateALT, ALAT Alanin aminotranspheraseALV Artificial lung ventilationAP Arterial (blood) pressureAST, AspAT Aspartat aminotranspheraseATP Adenosine triphosphateВСG Bacillus Calmette-GuérinBP Blood (arterial) pressureCK Creatine kinaseCNS Central nervous systemСоА Coenzyme ACRP C-reactive proteinCT Computed tomographyCOX CyclooxygenaseDIC Disseminated intravascular coagulationDTP (DPT) Diphtheria-tetanus-pertussis vaccineEDTA Ethylenediamine tetra-acetic acidELISA Enzyme-linked immunosorbent assayENT Ear, nose, and throat (as a department in a hospital)EPR (ER) Endoplasmic reticulumESR Erythrocyte sedimentation rateEV EnterovirusFC Functional classGABA Gamma aminobutyric acidHct, Ht HematocritHDLHR

High-density lipoproteinsHeart rate

IDL Intermediate-density lipoproteinsIU International unitIUPAC International Union of Pure and Applied ChemistryLDH Lactate dehydrogenaseLDL Low-density lipoproteinsLOX LipoxygenaseMAO Monoamine oxidaseMRI Magnetic resonance imagingNSAID Nonsteroidal anti-inflammatory drugPE (PATE) Pulmonary embolism (Pulmonary artery thromboembolism)PSA Prostate-specific antigenRBC Red blood countRR Respiratory rateSES Sanitary-and-epidemiologic stationSTD Sexually transmitted diseaseSTI Sexually transmitted infectionT/l Trillion/literTABT Typhoid-paratyphoid A and B plus tetanus toxoid vaccineTMJ Temporomandibular jointTSH Thyroid-stimulating hormoneTU Tuberculin unitURTI Upper respiratory tract infectionV/f Vision fieldVLDL Very-low-density lipoproteinsWBC White blood countWPW Wolff-Parkinson-White syndrome